Формула параллельных резисторов: Параллельное соединение резисторов | Онлайн калькулятор

Содержание

Последовательное и параллельное соединение резисторов


Последовательное соединение резисторов

Последовательное соединение резисторов применяется для увеличения сопротивления. Т.е. когда резисторы соединены последовательно, общее сопротивление равняется сумме сопротивлений каждого резистора. Например, если резисторы R1 и R2 соединены последовательно, их общее сопротивление высчитывается по формуле:
R = R1 + R2.
Это справедливо и для большего количества соединённых последовательно резисторов:
R = R1 + R2 + R3 + R4 + … + Rn.

 

Цепь из последовательно соединённых резисторов будет всегда иметь сопротивление большее, чем у любого резистора из этой цепи.

При последовательном соединении резисторов изменение сопротивления любого резистора из этой цепи влечёт за собой как изменение сопротивления всей цепи так и изменение силы тока в этой цепи.

Параллельное соединение резисторов (формула)

Параллельное соединение резисторов необходимо для уменьшения общего сопротивления и, как вариант, для увеличения мощности нескольких резисторов по сравнению с одним.

Расчет параллельного сопротивления

Расчет параллельного сопротивления двух параллельно соединённых резисторов R1 и R2 производится по следующей формуле:

 

Сопротивление из
двух резисторов:  
R = R1 × R2
 R1 + R2

Параллельное соединение трёх и более резисторов требует более сложной формулы для вычисления общего сопротивления:

Сопротивление параллельных резисторов

 1 
  =  
 1 + 1 + 1 + . ..
RR1R2R3

Как видно, вычислить сопротивление двух параллельных резисторов значительно удобнее.

Сопротивление параллельно соединённых резисторов будет всегда меньше, чем у любого из этих резисторов.

Параллельное соединение резисторов часто используют в случаях, когда необходимо сопротивление с большей мощностью. Для этого, как правило, используют резисторы с одинаковой мощностью и одинаковым сопротивлением. Общая мощность, в таком случае, вычисляется умножением мощности одного резистора на количество параллельно соединённых резисторов.
Например: десять резисторов номиналом 1 КОм и мощностью 1 Вт каждый, соединённые параллельно будут иметь общее сопротивление 100 Ом и мощность 10 Вт.
При последовательном соединении мощность резисторов также складывается. Т.е. в том же примере, но при последовательном соединении, общее сопротивление будет равно 10 КОм и мощность 10 Вт.

Параллельное соединение резисторов | Электротехника

Параллельное соединение резисторов. При параллельном соединении резисторов нескольких приемников они включаются между двумя точками электрической цепи, образуя

параллельные ветви (рис. 26, а). Заменяя

Рис. 26. Схемы параллельного соединения приемников

лампы резисторами с сопротивлениями R1, R2, R3, получим схему, показанную на рис. 26, б.
При параллельном соединении ко всем резисторам приложено одинаковое напряжение U. Поэтому согласно закону Ома:

I1=U/R1; I2=U/R2; I3=U/R3.

Ток в неразветвленной части цепи согласно первому закону Кирхгофа I = I1+I2+I3, или

I = U / R1 + U / R2 + U / R3 = U (1/R1 + 1/R2 + 1/R3) = U / Rэк (23)

Следовательно, эквивалентное сопротивление рассматриваемой цепи при параллельном соединении трех резисторов определяется формулой

1/Rэк

= 1/R1 + 1/R2 + 1/R3 (24)

Вводя в формулу (24) вместо значений 1/Rэк, 1/R1, 1/R2 и 1/R3 соответствующие проводимости Gэк, G1, G2 и G3, получим: эквивалентная проводимость параллельной цепи равна сумме проводимостей параллельно соединенных резисторов:

Gэк = G1+ G2 +G3 (25)

Таким образом, при увеличении числа параллельно включаемых резисторов результирующая проводимость электрической цепи увеличивается, а результирующее сопротивление уменьшается.
Из приведенных формул следует, что токи распределяются между параллельными ветвями обратно пропорционально их электрическим сопротивлениям или прямо пропорционально их проводимостям. Например, при трех ветвях

I1 : I2 : I3 = 1/R1 : 1/R2 : 1/R3 = G1 + G2 + G3 (26)

В этом отношении имеет место полная аналогия между распределением токов по отдельным ветвям и распределением потоков воды по трубам.
Приведенные формулы дают возможность определить эквивалентное сопротивление цепи для различных конкретных случаев. Например, при двух параллельно включенных резисторах результирующее сопротивление цепи

Rэк=R1R2/(R1+R2)

при трех параллельно включенных резисторах

Rэк=R1R2R3/(R1R2+R2R3+R1R3)

При параллельном соединении нескольких, например n, резисторов с одинаковым сопротивлением R1 результирующее сопротивление цепи Rэк будет в n раз меньше сопротивления R1, т.

е.

Rэк = R1 / n (27)

Проходящий по каждой ветви ток I1, в этом случае будет в п раз меньше общего тока:

I1 = I / n (28)

При параллельном соединении приемников, все они находятся под одним и тем же напряжением, и режим работы каждого из них не зависит от остальных. Это означает, что ток, проходящий по какому-либо из приемников, не будет оказывать существенного влияния на другие приемники. При всяком выключении или выходе из строя любого приемника остальные приемники остаются включенными. Поэтому параллельное соединение имеет существенные преимущества перед последовательным, вследствие чего оно получило наиболее широкое распространение. В частности, электрические лампы и двигатели, предназначенные для работы при определенном (номинальном) напряжении, всегда включают параллельно.
На электровозах постоянного тока и некоторых тепловозах тяговые двигатели в процессе регулирования скорости движения нужно включать под различные напряжения, поэтому они в процессе разгона переключаются с последовательного соединения на параллельное.

последовательное, параллельное, смешанное соединение. Расчет сопротивления

Резисторы между собой могут быть соединены двумя основными способами: последовательно и параллельно. Смешанное соединение резисторов является их комбинацией.

Сочетания любых соединений резисторов можно привести к одному резистору, расчетом сопротивления которого (R) мы сейчас займемся.

ПАРАЛЛЕЛЬНОЕ СОЕДИНЕНИЕ РЕЗИСТОРОВ

Давайте рассчитаем общее сопротивление такой цепи (рисунок 1). Для этого нам понадобится закон Ома — I=U/R и закон Кирхгофа — I=I1+I2+..In

С учетом этого имеем:

  • I=U/R
  • I1=U/R1
  • I2=U/R2
  • In=U/Rn
  • U/R=U/R1+U/R2+…U/Rn
  • 1/R=1/R
    1
    +1/R2+…1/Rn

Последняя формула является основной для расчета сопротивления цепи параллельно соединенных резисторов. Для двух резисторов ее можно записать более удобно: R=(R1*R2)/(R1+R2).

Отсюда следует, что в случае параллельного соединения двух одинаковых по номиналу резисторов (R1=R2) их общее сопротивление будет вдвое меньше любого из них. Это полезно помнить.

ПОСЛЕДОВАТЕЛЬНОЕ СОЕДИНЕНИЕ РЕЗИСТОРОВ

Используя уже упомянутые законы для цепи последовательно соединенных резисторов (рисунок 2) можем записать:

  • U=I*R
  • I=I1=I2=…In
  • U=U1+U2+…Un
  • I*R=I*R1+I*R2+…I*Rn
  • R=R1+R2+…Rn

То есть общее сопротивление резисторов при последовательном соединении равно сумме их сопротивлений.

СМЕШАННОЕ СОЕДИНЕНИЕ РЕЗИСТОРОВ

Такое соединение всегда можно представить как комбинацию последовательного и параллельного соединений (рис.3).

Расчет общего сопротивления цепи при этом производится поэтапно. В приведенном примере рассчитываем:

  1. последовательное сопротивление резисторов Rпосл=R1+R2
  2. параллельное соединение R=(Rпосл*R3)/(Rпосл+R3)

Безусловно, могут встретиться более сложные варианты, но методика расчета их сопротивления та же.

Несколько слов про то, когда возникает необходимость соединять резисторы тем или иным способом:

  1. Отсутствие «под рукой» резистора нужного номинала. При этом следует помнить, что погрешности резисторов будут суммироваться.

    Например, для рисунка 3.a, если фактическая погрешность R1 составляет +10%, а R2

    имеет +15%, то для Rпосл она будет +25%.

    Здесь следует обращать внимание на знак, то есть для -10% и +15% в результате получим +5%.

  2. Необходимость получить большую мощность.

    Здесь надо учесть, что при одинаковых номиналах сопротивлений и мощностей соединяемых резисторов, как при последовательном, так и при параллельном их соединении итоговая мощность будет равна сумме мощностей.

    В противном случае следует ее рассчитать, используя закон Ома и формулу для определения рассеиваемой мощности P=I*U.

Про мощность и номиналы резисторов можно почитать здесь.

© 2012-2020 г. Все права защищены.

Представленные на сайте материалы имеют информационный характер и не могут быть использованы в качестве руководящих и нормативных документов


Параллельное и последовательное соединение резисторов (сопротивлений)

Все разнообразие схем построено на двух типах соединения — параллельном и последовательном. Для разных соединений действуют разные законы, что и дает возможность создания устройств с различными характеристиками. Рассмотрим последовательное и параллельное соединение резисторов.

Содержание статьи

Что такое резистор и для чего он нужен

Резистор — это радиоэлемент, который увеличивает сопротивление цепи. Ставят его обычно для того, чтобы понизить/ограничить напряжение или ток. Есть сопротивления постоянные и переменные.

Например, светодиоды требуют небольшого тока, иначе перегревается и быстро выходит из строя. Чтобы ограничить ток, перед светодиодом поставьте сопротивление. Ток в цепи станет меньше.

Для чего нужны резисторы: для подстройки параметров питания

Постоянные сопротивления — это те, которые не меняют своего номинала в процессе работы. Если это и происходит, то считается выходом из строя.

Так выглядят переменные и постоянные резисторы

Переменные резисторы, наоборот, отличаются тем, что их сопротивление можно изменять. Они имеют бегунок или поворотную ручку, при помощи которых и изменяется номинал. На основе таких устройств делают регуляторы. Например, регулятор громкости, накала греющего элемента и т.д.

Последовательное соединение сопротивлений

Последовательное соединение характеризуется тем, что элементы идут друг за другом. Конец одного подключается к началу другого. При подключении полученной цепочки к источнику тока получается кольцо.

Лампы накаливания соединенные последовательно, можно рассматривать как сопротивления

Теоретическая часть

Последовательное соединение характерно тем, что через все элементы протекает ток одинаковой силы. То есть, если цепочка состоит из двух резисторов R1 и R2 (как на рисунке ниже), то ток протекающий через каждое из них и любую другую часть цепи будет одинаковой (I = I1 = I2).

Последовательно соединенные сопротивления. I1 — ток протекающий через резистор R1, I2 — ток протекающий через резистор R2

Суммарное сопротивление всей цепи последовательно соединенных резисторов считается как сумма сопротивлений всех ее элементов. То есть, номиналы складывают.

R = R1 + R2 — это и есть формула расчета сопротивления при последовательном соединении резисторов. Если элементов больше двух, будет просто больше слагаемых.

Еще одно свойство последовательного соединения — на каждом элементе напряжение отличается. Ток в цепи одинаковый, а напряжение на резисторе зависит от его номинала.

Примеры расчета

Давайте рассмотрим пример. Цепь представлена на рисунке выше. Есть источник тока и два сопротивления. Пусть R1=1,2 кОм, R2= 800 Ом, а ток в цепи 2 А. По закону Ома U = I * R. Подставляем наши значения:

  • U1 = R1 * I = 1200 Ом * 2 А = 2400 В;
  • U2 = R2 * I = 800 Ом * 2А = 1600 В.

Общее напряжение цепи считается как сумма напряжений на резисторах: U = U1 + U2 = 2400 В + 1600 В = 4000 В.

Так понятнее, что такое последовательное соединение

Полученную цифру можно проверить. Для этого найдем суммарное сопротивление цепи и умножим его на ток.   R = R1 + R2 = 1200 Ом + 800 Ом = 2000 Ом. Если подставить в формулу напряжения при последовательном соединении сопротивлений, получаем: U = R * I = 2000 Ом * 2  А = 4000 В. Получаем, что общее напряжение данной цепи 4000 В.

А теперь посмотрите на схему. На первом вольтметре (возле резистора R1) показания будут 2400 В, на втором  — 1600 В.  При этом напряжение источника питания — 4000 В.

Параллельное соединение резисторов

Параллельное соединение — это когда входы нескольких деталей соединяются в одной точке. Точно так же — в одну точку — соединяют их выходы.

Так выглядит параллельное соединение на схеме и в реальности

Теория и законы параллельного соединения

Если посмотреть на изображение параллельного соединения, заметно, что ко всем элементам прилагается одинаковое напряжение. То есть, при параллельном соединении резисторов, на каждом из них будет одинаковое напряжение.

U = U1 = U2 = U3.

Получается, что ток разделяется на несколько «ручейков». То есть, при параллельном соединении резисторов сила тока, протекающего через каждый из элементов, отличается. I = I1+I2+I3. И зависит сила тока (согласно тому же закону Ома) от сопротивления каждого участка цепи. В случае с параллельным соединением резисторов — от их номинала.

Так выглядит параллельное соединение резисторов на схеме

Общее сопротивление участка цепи при таком соединении становится ниже. Его высчитывают по формуле:

1/R = 1/R1 + 1/R + 1/R3+…

Такая форма хоть и понятна, но неудобна. Формула расчета сопротивления параллельно подключенных резисторов получается тем сложнее, чем больше элементов соединены параллельно. Но больше двух-трех редко кто объединяет, так что на практике достаточно знать только две формулы приведенные ниже.

Формулы расчета сопротивления при параллельном подключении двух и трех резисторов

Если подставить значения в эти формулы, то заметим, что результат будет меньше, чем сопротивление резистора с наименьшим номиналом. Это стоит запомнить: результирующее сопротивление включенных параллельно резисторов будет ниже самого маленького номинала.

Примеры расчета параллельного соединения сопротивлений

Давайте сначала рассчитаем параллельное соединение двух резисторов разного номинала и посмотрим что получится.

  • Соединили параллельно 150 Ом и 100 Ом. Считаем результирующее: 150*100 / (150+100) = 15000/250 = 60 Ом.
  • Если соединить 150 Ом и 50 Ом, получим: 150*50 / (150+50) = 7500 / 200 = 37,5 Ом.

Как видим, в обоих случаях результат оказывается меньше чем самый низкий номинал соединенных деталей. Этим и пользуются, если в наличии нет сопротивления небольшого номинала. Проблема только в том, что подбирать сложновато: надо каждый раз считать используя калькулятор.

Как высчитывать сопротивление составных резисторов

Возможно, вам будет проще, если знать, что соединив два одинаковых резистора параллельно, получим результат в два раза меньше. Например, соединив параллельно два резистора по 100 Ом получим составное сопротивление 50 Ом. Проверим? Считаем: 100*100 / (100+100) = 10000 / 200 = 50 Ом.

Еще один пример с лампочками

При соединении параллельно трех резисторов, считать приходится больше, так как формула сложнее. Но картина не отличается:

  • Если подключить параллельно 150 Ом, 100 Ом и 50 Ом, результирующее будет 27,3 Ом.
  • Попробуем с более низкими номиналами. Если параллельно включены 20 Ом, 15 Ом и 10 Ом. Получим результирующее сопротивление 4,61 Ом.

Вот вам подтверждение правила. Суммарное сопротивление параллельно соединенных резисторов меньше чем самый низкий номинал.

Смешанное соединение

Как быть, если в схеме есть и параллельное, и последовательное соединение резисторов? В таком случае считают общее сопротивление по участкам. Можно при этом перерисовывать схему, заменяя составные сопротивления на один «прямоугольник», но проставляя над ним высчитанный результат.

Пример расчета сопротивления при смешанном соединении резисторов. Рассматриваем исходную схему как совокупность параллельных и последовательных соединений

Шаг 1. Нашли общее сопротивление последовательно соединенных резисторов R3 и R4:

R3-4 = 3 кОм + 3 кОм = 6 кОм;

Шаг 2. Рассчитали сопротивление параллельно соединенных резисторов R2 и R3-4:

R2-4 = 3 кОм * 6 кОм / (3 кОм + 6 кОм) = 18 кОм/9 кОм = 2 кОм;

Шаг 3. Рассчитали общее сопротивление последовательно соединенных резисторов R1 и R2-4:

R1-4 = R1 + R2-4 = 1 кОм + 2 кОм = 3 кОм.

Практическое применение параллельного и последовательного соединения резисторов

Для чего практически можно использовать параллельное и последовательное соединение резисторов? Случается, что при ремонте электронной аппаратуры, не всегда в наличии сопротивление нужного номинала. Ехать в магазин за одним копеечным элементом — накладно. Вот тут и могут пригодиться составные резисторы. Просто надо последовательно или параллельно соединить их, подобрав требуемый номинал.

Последовательное и параллельное соединение резисторов применяют для подбора требуемого номинала. Контролировать точное значение получившегося сопротивления можно при помощи цифрового мультиметра

При соединении резисторов, их ножки первоначально скручивают. Какой стороной разворачивать сопротивление — неважно (в отличие от диодов, резисторы одинаково пропускают ток в обоих направлениях). На концах скрутку слегка обжимают плоскогубцами, затем пропаивают. Следите за тем, чтобы корпуса были друг от друга подальше — так они будут лучше охлаждаться при работе.

Как распределяются токи при параллельном соединении резисторов

Параллельным соединением сопротивлений называется такое соединение, при котором к одному зажиму источника подключаются начала сопротивлений, а к другому зажиму – концы.

Общее сопротивление параллельно включенных сопротивлений определяется по формуле

Общее сопротивление параллельно включенных сопротивлений всегда меньше наименьшего сопротивления, входящего в данное соединение.

На вышеуказанном рисунке мы можем сразу сказать что общее сопротивление будет меньше 10 ом.

Первый частный случай

Если параллельно включено только два резистора то их общее сопротивление можно определить по формуле

Второй частный случай

Если параллельно включено любое количество резисторов одинаковых сопротивлений то их общее сопротивление можно определить если сопротивление одного резистора разделить на количество резисторов.

Так как начала всех сопротивлений сведены в одну общую точку, а концы – в другую, то очевидно, что разность потенциалов на концах любого из параллельно включенных сопротивлений равна разности потенциалов между общими точками.

Итак, при параллельном соединении сопротивлений напряжения на них равны между собой.

Если разветвление подключено непосредственно к зажимам источника тока, то напряжение на каждом из сопротивлений равно напряжению на зажимах источника.

Второе свойство цепи с параллельным соединением заключается в том, что электрический ток распределяется по параллельным ветвям обратно пропорционально их сопротивлениям.

Это значит что чем больше сопротивление тем меньше по нему пойдет ток.

Рассматривая точку разветвления А, замечаем, что к ней притекает ток I, а токи I1, I2, I3 утекают из нее. Так как движущиеся электрические заряды не скапливаются в точке, то очевидно, что суммарный заряд, притекающий к точке разветвления, равен суммарному заряду утекающему от нее:

Следовательно, третье свойство параллельного соединения может сформулирована так:

Величина тока в не разветвленной части цепи равна сумме токов в параллельных ветвях.

Последовательное соединение резисторов

Последовательное соединениеэто соединение двух или более резисторов в форме цепи, в которой каждый отдельный резистор соединяется с другим отдельным резистором только в одной точке.

Общее сопротивление R

общ

При таком соединении, через все резисторы проходит один и тот же электрический ток. Чем больше элементов на данном участке электрической цепи, тем «труднее» току протекать через него. Следовательно, при последовательном соединении резисторов их общее сопротивление увеличивается, и оно равно сумме всех сопротивлений.

Напряжение при последовательном соединении

Напряжение при последовательном соединении распределяется на каждый резистор согласно закону Ома:

Т.е чем большее сопротивление резистора, тем большее напряжение на него падает.

Параллельное соединение резисторов

Параллельное соединениеэто соединение, при котором резисторы соединяются между собой обоими контактами. В результате к одной точке (электрическому узлу) может быть присоединено несколько резисторов.

Общее сопротивление R

общ

При таком соединении, через каждый резистор потечет отдельный ток. Сила данного тока будет обратно пропорциональна сопротивлению резистора. В результате общая проводимость такого участка электрической цепи увеличивается, а общее сопротивление в свою очередь уменьшается.

Таким образом, при параллельном подсоединении резисторов с разным сопротивлением, общее сопротивление будет всегда меньше значения самого маленького отдельного резистора.

Формула общей проводимости при параллельном соединении резисторов:

Формула эквивалентного общего сопротивления при параллельном соединении резисторов:

Для двух одинаковых резисторов общее сопротивление будет равно половине одного отдельного резистора:

Соответственно, для n одинаковых резисторов общее сопротивление будет равно значению одного резистора, разделенного на n.

Напряжение при параллельном соединении

Напряжение между точками A и B является как общим напряжением для всего участка цепи, так и напряжением, падающим на каждый резистор в отдельности. Поэтому при параллельном соединении на все резисторы упадет одинаковое напряжение.

Электрический ток при параллельном соединении

Через каждый резистор течет ток, сила которого обратно пропорциональна сопротивлению резистора. Для того чтобы узнать какой ток течет через определенный резистор, можно воспользоваться законом Ома:

Смешанное соединение резисторов

Смешанным соединением называют участок цепи, где часть резисторов соединяются между собой последовательно, а часть параллельно. В свою очередь, смешанное соединение бывает последовательного и параллельного типов.

Общее сопротивление R

общ

Для того чтобы посчитать общее сопротивление смешанного соединения:

  • Цепь разбивают на участки с только пареллельным или только последовательным соединением.
  • Вычисляют общее сопротивление для каждого отдельного участка.
  • Вычисляют общее сопротивление для всей цепи смешанного соединения.

Так это будет выглядеть для схемы 1:

Также существует более быстрый способ расчета общего сопротивления для смешанного соединения. Можно, в соответствии схеме, сразу записывать формулу следующим образом:

  • Если резисторы соединяются последоватеьно — складывать.
  • Если резисторы соединяются параллельно — использовать условное обозначение «||».
  • Подставлять формулу для параллельного соединения где стоит символ «||».

Так это будет выглядеть для схемы 1:

После подстановки формулы параллельного соединения вместо «||»:

Параллельным соединением резисторов (или приемников энергии, ветвей,сопротивлений) называется такое, при котором к одним и тем же двум узлам электрической цепи (рисунок 1) присоединены несколько резисторов (ветвей).

Рис. 1 Изображение параллельного соединения трех резисторов

Проводимость при параллельном соединении

Сопротивление при параллельном соединении:

Для трёх параллельно соединенных сопротивлений

Для двух параллельно соединенных сопротивлений

Для ветвей с одинаковым сопротивлением где n количество ветвей

Ток при параллельном соединении

Мощность при параллельном соединении

Доказательство

Так как резисторы присоединены к одним и тем же узлам, то каждый из них находится под одинаковым напряжением U. Согласно закону Ома токи в сопртивлениях определяются по формулам

Из этих формул следует, что токи в параллельных ветвях с сопротивлениями распределяются прямо пропорционально проводимостям ветвей или обратно пропорционально их сопротивлениям. Ряд параллельно соединенных резисторов можно заменить эквивалентным с сопротивлением R, значение которого должно быть таким, чтобы при том же напряжении на выводах ток в эквивалентном резисторе был равен сумме токов в отдельных ветвях:

т. е. эквивалентная проводимость параллельного соединения резисторов равна сумме проводимостей всех параллельных ветвей. Следовательно, эквивалентное сопротивление будет меньше самого малого из параллельно соединенных резисторов.
Формула (1) дает возможность определить и эквивалентное сопротивление параллельного соединения резисторов. Например, при трех ветвях эквивалентная проводимость

и эквивалентное сопротивление

Для двух резисторов

Если сопротивление ветвей одинаково R1 = R2 = R3, то можно воспользоваться формулой

в общем случае при соединении n резисторов с одинаковым сопротивлением R1 эквивалентное сопротивление равно

Мощности параллельно соединенных резисторов равна сумме мощностей всех резисторов

Расчет сопротивления двух параллельно соединенных резисторов. Последовательное и параллельное соединение резисторов. Формула для расчета параллельного соединения сопротивлений

В каждой электрической схеме присутствует резистор, имеющий сопротивление электрическому току. Резисторы бывают двух типов: постоянные и переменные. Во время разработки любой электрической схемы и ремонта электронных изделий часто приходится применять резистор, обладающий необходимым номиналом.

Несмотря на то что для резисторов предусмотрены различные номиналы , может случиться так, что не будет возможности найти необходимый или же вообще ни один элемент не сможет обеспечить требуемый показатель.

Рассчитать производительность и работу

Угол сдвига фаз вычисляется по изображению указателя. Чтобы иметь возможность определять мощности, поглощаемые схемой, предыдущие формулы используются снова. Для определения работы используются следующие формулы.

Дальнейший интересный контент по теме
Резисторы переменного тока представляют собой омические, индуктивные и емкостные резисторы. Для параллельного подключения таких резисторов в цепи переменного тока применяются разные законы, чем для сопротивлений в цепи постоянного тока. Учитывая это, например, катушку: настоящая катушка имеет как индуктивное, так и омическое сопротивление и поэтому может рассматриваться как последовательная связь чисто индуктивного и чисто омического резистора.

Решением этой проблемы может стать применение последовательного и параллельного соединения. Ознакомившись с этой статьей, вы узнаете об особенностях выполнения расчета и подбора различных номиналов сопротивлений.

Часто при изготовлении какого-либо устройства используют резисторы, которые соединяются в соответствии с последовательной схемой. Эффект от применения такого варианта сборки сводится к увеличению общего сопротивления цепи. Для данного варианта соединения элементов создаваемое ими сопротивление рассчитывается как сумма номиналов. Если же сборка деталей выполняется по параллельной схеме, то здесь потребуется рассчитать сопротивление , используя нижеописанные формулы.

Примеры параллельного соединения проводников

Мы рассматриваем только индуктивную составляющую резистора, т.е. катушку как чисто индуктивный резистор. Аналогично, используется омическое сопротивление и емкостное сопротивление, поскольку омическое сопротивление также может иметь индуктивный компонент. В то время как в случае сопротивления проволоки витки, подобные виткам, видны напрямую, это обычно скрыто в резисторах слоя. Фактически, проводящий слой наносят на носитель, из которого материал, проводящий материал, удаляется с помощью процесса спирально-циркулирующего фрезерования, так что остается спирально циркулирующий слой.

К схеме параллельного соединения прибегают в ситуации, когда стоит задача по снижению суммарного сопротивления, а, помимо этого, увеличения мощности для группы элементов, подключенных по параллельной схеме, которое должно быть больше, чем при их отдельном подключении.

Таким образом генерируется требуемое значение сопротивления. Сразу видно, что эта катушечная структура приводит к индуктивному компоненту. Однако это обычно настолько мало, что его можно пренебречь. Общая обработка взаимосвязи любых резисторов переменного тока невозможна и не требуется с помощью математических знаний, доступных в школе.

Объяснение Подключение серии и параллельное соединение

Ниже приведен упрощенный случай параллельной схемы чисто омического, индуктивного и емкостного резисторов. В этой статье мы рассмотрим параллельное соединение и последовательное соединение резисторов. Давайте сначала уточним, что такое последовательное соединение и что такое параллельное соединение, и где разница между последовательным соединением и параллельным соединением. В последовательной цепи мы имеем два или более сопротивления последовательно. Тот же ток протекает через все резисторы.

Расчет сопротивления

В случае подключения деталей друг с другом, с применением параллельной схемы для расчета суммарного сопротивления, будет использоваться следующая формула:

R(общ)=1/(1/R1+1/R2+1/R3+1/Rn).

  • R1- R3 и Rn – резисторы, подсоединенные по параллельной схеме.

Причем, если цепь создается на основе только двух элементов, то для определения суммарного номинального сопротивления следует использовать такую формулу:

Универсальная схема расчета

На следующем графике показаны резисторы последовательно, два резистора, индивидуально нарисованные в начале, и три резистора под электрической цепью. Напротив, существует параллельное соединение резисторов. Что такое параллельная схема? Теперь, в параллельной цепи, линия распадается, и, следовательно, и ток разлагается. В случае параллельного подключения резисторов во многих случаях впервые рассматривается параллельное соединение двух резисторов. Это выглядит следующим образом, включая формулу для расчета.

R(общ)=R1*R2/R1+R2.

  • R(общ) – суммарное сопротивление;
  • R1 и R2 – резисторы, подсоединенные по параллельной схеме.

Универсальная схема расчета

Применительно к радиотехнике следует уделить внимание одному важному правилу: если подключаемые друг к другу элементы по параллельной схеме имеют одинаковый показатель , то для расчета суммарного номинала необходимо общее значение разделить на число подключенных узлов:

Для трех резисторов в параллельной схеме это будет выглядеть на следующем графике, включая формулу для расчета. Разностное соединение и параллельное соединение. В случае последовательной цепи все резисторы подключаются по одной линии за другой. В случае параллельной схемы, с другой стороны, линия расщепляется, резисторы лежат в отдельных линиях. В последовательной цепи тот же ток протекает через все резисторы, а в случае параллельной цепи ток расщепляется. В случае параллельной схемы одно и то же напряжение подается на каждый резистор, но не в последовательной цепи. Другое примечание: смесь последовательной цепи и параллельной схемы называется групповой схемой.

  • R(общ) – суммарное значение сопротивления;
  • R – номинал резистора, подсоединенного по параллельной схеме;
  • n – число подключенных узлов.

Особое внимание следует обратить на то, что конечный показатель сопротивления в случае использования параллельной схемы подключения обязательно будет меньше по сравнению с номиналом любого элемента, подключаемого в цепь.

Примеры Подключение серий и параллельное соединение

В следующих примерах мы увидим, как рассчитать смесь схемы параллельной цепи и серии. В этой области шаг за шагом должен быть рассчитан набор последовательных схем и параллельных схем. На следующем графике показана смесь последовательного соединения и параллельного соединения. Каково общее сопротивление?

Сначала мы суммируем 20 Ом и 30 Ом, так как здесь имеется параллельная схема. Таким образом, схема выглядит следующим образом. Теперь добавим эту схему, добавив резисторы для вычисления общего сопротивления. Вычислите общее сопротивление следующего контура.

Пример расчета

Для большей наглядности можно рассмотреть следующий пример: допустим, у нас есть три резистора, чьи номиналы соответственно равны 100, 150 и 30 Ом. Если воспользоваться первой формулой для определения общего номинала, то получим следующее:

Прежде всего, вы должны увидеть, что есть короткое замыкание на резисторе с 95 Ом. Поэтому ток течет практически полностью по линии ниже, а 95 Ом не учитывается при расчете полного сопротивления. В противном случае у нас есть сочетание последовательной цепи и параллельной схемы.

Комбинированные последовательные и параллельные схемы

Вам нужно знать, как рассчитать резисторы последовательно, параллельно и комбинацию резисторов параллельно и последовательно? Если вы не хотите жарить свою печатную плату, вы это делаете! Эта статья покажет вам, как это сделать за несколько простых шагов. Это просто образный способ говорить, чтобы понятий было легко понять.

Некоторые факты, которые вы должны учитывать
Любой материал, который проводит электрический ток, имеет удельное сопротивление, которое представляет собой сопротивление материала при прохождении электрического тока.
  • Понять понятие сопротивления.
  • Единицей измерения резисторов является Ом.
Параллельное соединение резисторов характеризуется тем, что входные клеммы каждого из резисторов соединены друг с другом.

R(общ)=1/(1/100+1/150+1/30)=1/(0,01+0,007+0,03)=1/0,047=21,28Ом.

Если выполнить несложные расчеты, то можно получить следующее: для цепи, включающей в себя три детали, где наименьший показатель сопротивления составляет 30 Ом, результирующее значение номинала будет равно 21,28 Ом. Этот показатель будет меньше минимального значения номинала в цепи практически на 30%.

Аналогично, в конфигурации параллельного резистора выходные клеммы также соединены друг с другом. Из-за этого все резисторы пропускают одинаковое напряжение, т.е. имеют одинаковое падение напряжения. Это связано с тем, что концы каждого из резисторов соединены с одной и той же точкой в ​​цепи, и поэтому они имеют одинаковое напряжение.

Однако общий ток, протекающий через резисторы параллельно, равен сумме интенсивностей, которые проходят каждый резистор. Дифференциация параллельного соединения из последовательного интерфейса проста. В последовательной конфигурации резистора выходной разъем одного подключается к входному разъему следующего.

Важные нюансы

Обычно для резисторов параллельное соединение применяется тогда, когда стоит задача по созданию сопротивления большей мощности. Для ее решения потребуются резисторы, которые должны иметь равные показатели сопротивления и мощности. При таком варианте определить общую мощность можно следующим образом : мощность одного элемента необходимо перемножить с суммарным числом всех резисторов, из которых состоит цепь, подсоединенных друг с другом в соответствии с параллельной схемой.

Вычисление сопротивлений параллельно: формула

Чтобы вычислить эквивалентное сопротивление нескольких подключенных параллельных резисторов, мы должны применить формулу, указанную выше этих строк. Чтобы избежать ошибок в расчетах, лучше всего разделить формулу на два шага. Сначала мы вычисляем сумму обратного для каждого сопротивления и, когда получаем результат, вычисляем его обратно, чтобы знать эквивалентное сопротивление.

Решенное сопротивление сопротивлениям параллельно

Например, мы вычислим эквивалентное сопротивление конфигурации, аналогичное той, что мы имеем на следующем рисунке. Первый шаг: вычислить сумму обратного каждого сопротивления. Шаг второй: вычислите обратное только что полученное сопротивление.

Вычисление трех резисторов параллельно
Если мы хотим решить предыдущий пример, но используя наш калькулятор из трех резисторов параллельно в сети, просто заполните значение каждого резистора в соответствующем поле. Порядок, в котором вы его пишете, не имеет значения, поэтому вам не нужно его уважать.

Скажем, если нами будут использоваться пять резисторов, чей номинал составляет 100 Ом, а мощность каждого равна 1 Вт, которые присоединены друг к другу в соответствии с параллельной схемой, то суммарный показатель сопротивления будет равен 20 Ом, а мощность составит 5 Вт.

Если взять те же резисторы, но подсоединить их в соответствии с последовательной схемой, то конечная мощность составит 5 Вт, а суммарный номинал будет равен 500 Ом.

Когда вы пишете значение трех резисторов параллельно, просто нажмите кнопку расчета, и вы автоматически получите результат без применения формулы для расчета сопротивления параллельно. С этим вы экономите время и, прежде всего, просчеты. Как мы видели в предыдущих разделах, устройства, которые выступают против прохода электрического тока более выраженным образом, чем обычно, обычно используются в электрических цепях. Эти устройства называются резисторами и могут быть связаны таким образом, что вместе они эквивалентны значению другого сопротивления, называемого эквивалентным сопротивлением.

Заключение

Параллельная схема подключения резисторов очень востребована по той причине, что часто возникает задача по созданию такого номинала, которого невозможно добиться при помощи простого параллельного соединения. При этом процедура расчета этого параметра отличается достаточной сложностью , где необходимо учитывать разные параметры.

Ассоциация резисторов в серии

Он называется полученным сопротивлением или эквивалентом, к значению сопротивления, которое получается путем связывания их набора. В основном резисторы могут быть связаны последовательно, параллельно или комбинацией обоих смешанных вызовов. Когда два или более резисторов последовательно, интенсивность тока, проходящая через каждую из них, одинакова.

Если применить закон Ома к каждому из сопротивлений предыдущего рисунка, мы получим. Если мы сделаем сумму от члена к элементу по трем уравнениям, заметим, что. Таким образом, приведенное выше уравнение, если учесть, что. Итак, если вы понимаете, вы можете видеть, что три предыдущих резистора серии эквивалентны одному резистору, значение которого представляет собой сумму трех предыдущих.

Здесь важная роль отводится не только количеству подключаемых элементов, но и рабочим параметрам резисторов — прежде всего, сопротивлению и мощности. Если один из подключаемых элементов будет иметь неподходящий показатель, то это не позволит эффективно решить задачу по созданию требуемого номинала в цепи.

Последовательное соединение это соединение двух или более резисторов в форме цепи, в которой каждый отдельный резистор соединяется с другим отдельным резистором только в одной точке.

Ассоциация резисторов в параллельном

Когда два или более резисторов параллельны, они делятся своими концами, как показано на следующем рисунке. Если мы применим закон Ома в каждом из сопротивлений фигуры. Зная, что сумма интенсивностей каждого сопротивления равна интенсивности перед входом и выходом из набора, образованного тремя сопротивлениями.

Ассоциация смешанного сопротивления

Как правило, в электрических цепях они не просто похожи на последовательные или параллельные резисторы, но и на комбинацию обоих. Чтобы лучше понять, как подойти к этим типам ассоциаций, мы проиллюстрируем пример. Представьте себе следующую схему сопротивлений.

Общее сопротивление R общ

При таком соединении, через все резисторы проходит один и тот же электрический ток. Чем больше элементов на данном участке электрической цепи, тем «труднее» току протекать через него. Следовательно, при последовательном соединении резисторов их общее сопротивление увеличивается, и оно равно сумме всех сопротивлений.

Подключение 2 равных громкоговорителей последовательно добавляет импедансы и ватты? Сопротивление добавляется, и общая мощность рассеивания на громкоговоритель уменьшается вдвое. Предположим, что выход 8 вольт и динамик 8 Ом ток, который циркулирует громкоговорителем.

Таким образом, динамик должен будет поддерживать более 8 Вт, чтобы он не был поврежден. Теперь подключите два динамика 8 Ом и 8 Вт последовательно, импеданс обоих составляет 16 Ом. Теперь мы вычисляем ток, протекающий через динамики. С этими данными мы вычисляем мощность в каждом динамике.

Напряжение при последовательном соединении

Напряжение при последовательном соединении распределяется на каждый резистор согласно закону Ома:

Т.е чем большее сопротивление резистора, тем большее напряжение на него падает.

Вывод: динамики работают более сдержанно, но усилитель будет поставлять половину мощности, когда динамик составляет 8 Ом. Разделен ли импеданс и ватт параллельно? Если мы рассмотрим одни и те же ораторы на примере предыдущего вопроса, можно сказать, что общий импеданс уменьшается наполовину и рассчитывается следующим образом.

Для работы с более чем двумя динамиками параллельно необходимо использовать другое уравнение для расчета импеданса. Если мы анализируем отдельно каждого динамика, мы понимаем, что каждый из них ведет себя так же, как в примере 1, когда каждый из них подключен к усилителю, через который они будут циркулировать 1 А, а мощность, подлежащая рассеиванию, будет составлять 8 Вт в каждом динамике.

Параллельное соединение это соединение, при котором резисторы соединяются между собой обоими контактами. В результате к одной точке (электрическому узлу) может быть присоединено несколько резисторов.

Общее сопротивление R общ

При таком соединении, через каждый резистор потечет отдельный ток. Сила данного тока будет обратно пропорциональна сопротивлению резистора. В результате общая проводимость такого участка электрической цепи увеличивается, а общее сопротивление в свою очередь уменьшается.

Таким образом, при параллельном подсоединении резисторов с разным сопротивлением, общее сопротивление будет всегда меньше значения самого маленького отдельного резистора.

Формула общей проводимости при параллельном соединении резисторов:

Формула эквивалентного общего сопротивления при параллельном соединении резисторов:

Для двух одинаковых резисторов общее сопротивление будет равно половине одного отдельного резистора:

Соответственно, для n одинаковых резисторов общее сопротивление будет равно значению одного резистора, разделенного на n.

Напряжение при параллельном соединении

Напряжение между точками A и B является как общим напряжением для всего участка цепи, так и напряжением, падающим на каждый резистор в отдельности. Поэтому при параллельном соединении на все резисторы упадет одинаковое напряжение.

Через каждый резистор течет ток, сила которого обратно пропорциональна сопротивлению резистора. Для того чтобы узнать какой ток течет через определенный резистор, можно воспользоваться законом Ома:

Смешанным соединением называют участок цепи, где часть резисторов соединяются между собой последовательно, а часть параллельно. В свою очередь, смешанное соединение бывает последовательного и параллельного типов.

Общее сопротивление R общ

  • Цепь разбивают на участки с только пареллельным или только последовательным соединением.
  • Вычисляют общее сопротивление для каждого отдельного участка.
  • Вычисляют общее сопротивление для всей цепи смешанного соединения.

Также существует более быстрый способ расчета общего сопротивления для смешанного соединения. Можно, в соответствии схеме, сразу записывать формулу следующим образом:

  • Если резисторы соединяются последоватеьно — складывать.
  • Если резисторы соединяются параллельно — использовать условное обозначение «||».
  • Подставлять формулу для параллельного соединения где стоит символ «||».

Так это будет выглядеть для схемы 1:

Параллельное и последовательное соединение резисторов

Автор Alexey На чтение 5 мин. Просмотров 331 Опубликовано Обновлено

В электротехнике и электронике очень широко используются резисторы. Применяются они в основном для регулирования в схемах тока и напряжения. Основные параметры : электрическое сопротивление (R) измеряется в Омах, мощность (Вт) , стабильность и точность их параметров  в процессе эксплуатации. Можно вспомнить ещё множество его параметров , — ведь это обычное промышленное изделие.

Последовательное соединение

Последовательное соединение  — это такое соединение, при котором каждый последующий резистор подключается к предыдущему, образуя неразрывную цепь без разветвлений. Ток I=I1=I2 в такой цепи будет одинаковым в каждой её точке. Напротив, напряжение U1, U2 в различных её точках будет разным, причём работа по переносу заряда через всю цепь, складывается из работ по переносу заряда в каждом из резисторов, U=U1+U2. Напряжение U по закону Ома равно току, умноженному на сопротивление, и предыдущее выражение можно записать так:

IR=IR1+IR2,

где R — общее сопротивление цепи. То есть по простому идет падение напряжения в точках соединения резисторов и чем больше подключенных элементов , тем больше происходит падение напряжения

Отсюда следует, что  , общее значение  такого соединения определяется суммированием сопротивлений последовательно . Наши рассуждения справедливы для любого количества последовательно соединяемых участков цепи.

Параллельное соединение

Объединим начала нескольких резисторов (точка А). В другой точке (В) мы соединим все их концы. В результате получим участок цепи, который называется параллельным соединением и состоит из некоторого количества параллельных друг другу ветвей (в нашем случае – резисторов). При этом электрический ток между точками А и B распределится по каждой из этих ветвей.

Напряжения на всех резисторах будут одинаковы: U=U1=U2=U3, их концы — это точки А и В.

Заряды, прошедшие за единицу времени через каждый резистор, в сумме образуют заряд, прошедший через весь блок. Поэтому суммарный ток через изображенную на рисунке цепь I=I1+I2+I3.

Теперь, использовав закон Ома, последнее равенство преобразуется к такому виду:

U/R=U/R1+U/R2+U/R3.

Отсюда следует, что для эквивалентного сопротивления R справедливо:

1/R=1/R1+1/R2+1/R3

или после преобразования формулы мы можем получить другую запись, такого вида : .

Чем большее количество резисторов (или других звеньев электрической цепи, обладающих некоторым сопротивлением) соединить по параллельной схеме, тем больше путей для протекания тока образуется, и тем меньше общее сопротивление цепи.

Следует отметить, что обратная сопротивлению величина называется проводимостью. Можно сказать, что при параллельном соединении участков цепи складываются проводимости этих участков, а при последовательном соединении – их сопротивления.

Примеры использования

Понятно, что при последовательном соединении, разрыв цепи в одном месте приводит к тому, что ток перестает идти по всей цепи. Например, ёлочная гирлянда перестаёт светить, если перегорит всего одна лампочка, это плохо.

Но последовательное соединение лампочек в гирлянде даёт возможность использовать большое количество маленьких лампочек, каждая из которых рассчитана на напряжение сети (220 В), делённое на количество лампочек.

Последовательное соединение резисторов на примере 3-х лампочек и ЭДС

Зато при последовательном подключении предохранительного устройства его срабатывание (разрыв плавкой вставки) позволяет обесточить всю электрическую цепь, расположенную после него и обеспечить нужный уровень безопасности, и это хорошо. Выключатель в сеть питания электроприбора включается также последовательно.

Параллельное соединение также широко используется. Например, люстра – все лампочки соединены параллельно и находятся под одним и тем же напряжением. Если одна лампа перегорит, — не страшно, остальные не погаснут, они остаются под тем же самым напряжением.

Параллельное соединение резисторов на примере 3-х лампочек и генератора

При необходимости увеличения способности схемы рассеивать тепловую мощность, выделяющуюся при протекании тока, широко используются и последовательное, и параллельное объединение резисторов. И для последовательного, и параллельного способов соединения некоторого количества резисторов одного номинала общая мощность равна произведению количества резисторов на мощность одного резистора.

Смешанное соединение резисторов

Также часто используется смешанное соединение . Если ,например необходимо получить сопротивление  определенного номинала, но его нет в наличии можно воспользоваться одним из выше описанных способов или воспользоваться смешанным соединением.

Отсюда , можно вывести формулу которая и даст нам необходимое значение:

Rобщ.=(R1*R2/R1+R2)+R3

В нашу эпоху развития электроники и различных технических устройств в основе всех сложностей лежать простые  законы, которые поверхностно рассматриваются на данном сайте и думаю, что вам они помогут успешно применять в своей жизни. Если например взять ёлочную гирлянду , то соединения лампочек идет друг за другом , т.е. грубо говоря это отдельно-взятое сопротивление.

Не так давно гирлянды стали соединятся смешанным способом. Вообще , в совокупности все эти примеры с резисторами взяты условно , т.е. любым элементом сопротивления может быть  ток проходящий через элемент с падением напряжения и выделением тепла.

резисторов последовательно и параллельно

Цели обучения

К концу этого раздела вы сможете:

  • Нарисуйте цепь с резисторами, включенными параллельно и последовательно.
  • Рассчитайте падение напряжения тока на резисторе, используя закон Ома.
  • Контраст — способ расчета общего сопротивления для резисторов, включенных последовательно и параллельно.
  • Объясните, почему полное сопротивление параллельной цепи меньше наименьшего сопротивления любого из резисторов в этой цепи.
  • Вычислить общее сопротивление цепи, которая содержит смесь резисторов, соединенных последовательно и параллельно.

Большинство схем имеет более одного компонента, называемого резистором , который ограничивает поток заряда в цепи. Мера этого предела расхода заряда называется сопротивлением . Простейшие комбинации резисторов — это последовательное и параллельное соединение, показанное на рисунке 1. Общее сопротивление комбинации резисторов зависит как от их индивидуальных значений, так и от способа их подключения.

Рис. 1. (a) Последовательное соединение резисторов. (б) Параллельное соединение резисторов.

Когда резисторы в серии ? Резисторы включены последовательно всякий раз, когда поток заряда, называемый током , должен проходить через устройства последовательно. Например, если ток течет через человека, держащего отвертку, в землю, тогда R 1 на Рисунке 1 (а) может быть сопротивлением вала отвертки, R 2 сопротивлением ее ручки , R 3 сопротивления тела человека и R 4 сопротивления его обуви.На рисунке 2 показаны резисторы, последовательно подключенные к источнику напряжения . Кажется разумным, что полное сопротивление является суммой отдельных сопротивлений, учитывая, что ток должен проходить через каждый резистор последовательно. (Этот факт был бы преимуществом для человека, желающего избежать поражения электрическим током, который мог бы уменьшить ток, надев обувь с резиновыми подошвами с высоким сопротивлением. прибор, уменьшающий рабочий ток.)

Рис. 2. Три резистора, подключенных последовательно к батарее (слева), и эквивалентное одиночное или последовательное сопротивление (справа).

Чтобы убедиться, что последовательно соединенные сопротивления действительно складываются, давайте рассмотрим потерю электроэнергии, называемую падением напряжения , в каждом резисторе на рисунке 2. Согласно закону Ома, падение напряжения, В, , на резистор, когда через него протекает ток, рассчитывается по формуле V = IR , где I равно току в амперах (A), а R — сопротивление в омах (Ω).Другой способ представить это: В, — это напряжение, необходимое для протекания тока I через сопротивление R . Таким образом, падение напряжения на R 1 составляет В 1 = IR 1 , что на R 2 составляет В 2 = IR 2 и что для R 3 это V 3 = IR 3 .Сумма этих напряжений равна выходному напряжению источника; то есть

В = В 1 + В 2 + В 3 .

Это уравнение основано на сохранении энергии и сохранении заряда. Электрическая потенциальная энергия может быть описана уравнением PE = qV , где q — электрический заряд, а V — напряжение. Таким образом, энергия, отдаваемая источником, составляет кв.кв. , а энергия, рассеиваемая резисторами, составляет

.

qV 1 + qV 2 + qV 3 .

Установление связей: законы сохранения

Вывод выражений для последовательного и параллельного сопротивления основан на законах сохранения энергии и сохранения заряда, которые утверждают, что общий заряд и полная энергия постоянны в любом процессе. Эти два закона непосредственно участвуют во всех электрических явлениях и будут многократно использоваться для объяснения как конкретных эффектов, так и общего поведения электричества.

Эти энергии должны быть равны, потому что в цепи нет другого источника и другого назначения для энергии.Таким образом, qV = qV 1 + qV 2 + qV 3 . Плата q аннулируется, давая V = V 1 + V 2 + V 3 , как указано. (Обратите внимание, что одинаковое количество заряда проходит через батарею и каждый резистор за заданный промежуток времени, поскольку нет емкости для хранения заряда, нет места для утечки заряда и заряд сохраняется.) Теперь подстановка значений для отдельных напряжений дает

В = IR 1 + IR 2 + IR 3 = I ( R 1 + R 2 + R 3 ).

Обратите внимание, что для эквивалентного сопротивления одной серии R с , мы имеем

В = ИК с .

Это означает, что полное или эквивалентное последовательное сопротивление R с трех резисторов составляет R с = R 1 + R 2 + R 3 .Эта логика действительна в общем для любого количества резисторов, включенных последовательно; таким образом, полное сопротивление R с последовательного соединения составляет

R с = R 1 + R 2 + R 3 +…,

, как предлагается. Поскольку весь ток должен проходить через каждый резистор, он испытывает сопротивление каждого, а последовательно соединенные сопротивления просто складываются.

Пример 1. Расчет сопротивления, тока, падения напряжения и рассеиваемой мощности: анализ последовательной цепи

Предположим, что выходное напряжение батареи на рисунке 2 равно 12.0 В, а сопротивления равны R 1 = 1,00 Ом, R 2 = 6,00 Ом и R 3 = 13,0 Ом. а) Каково полное сопротивление? (б) Найдите ток. (c) Рассчитайте падение напряжения на каждом резисторе и покажите, как они складываются, чтобы равняться выходному напряжению источника. (d) Рассчитайте мощность, рассеиваемую каждым резистором. (e) Найдите выходную мощность источника и покажите, что она равна общей мощности, рассеиваемой резисторами.

Стратегия и решение для (а)

Общее сопротивление — это просто сумма отдельных сопротивлений, определяемая следующим уравнением:

[латекс] \ begin {array} {lll} {R} _ {\ text {s}} & = & {R} _ {1} + {R} _ {2} + {R} _ {3} \ \ & = & 1.00 \ text {} \ Omega + 6.00 \ text {} \ Omega + 13.0 \ text {} \ Omega \\ & = & 20.0 \ text {} \ Omega \ end {array} \\ [/ latex].

Стратегия и решение для (b)

Ток определяется по закону Ома: В = IR . Ввод значения приложенного напряжения и общего сопротивления дает ток для цепи:

[латекс] I = \ frac {V} {{R} _ {\ text {s}}} = \ frac {12.0 \ text {V}} {20.0 \ text {} \ Omega} = 0.60 \ text {A }\\[/латекс].

Стратегия и решение для (c)

Напряжение — или падение IR — на резисторе определяется законом Ома.Ввод тока и значения первого сопротивления дает

.

В 1 = IR 1 = (0,600 A) (1,0 Ом) = 0,600 В.

Аналогично

В 2 = IR 2 = (0,600 A) (6,0 Ом) = 3,60 В

и

V3 = IR 3 = (0,600 A) (13,0 Ом) = 7,80 В.

Обсуждение для (c)

Три капли IR добавляют к 12.0 В, прогноз:

В 1 + В 2 + В 3 = (0,600 + 3,60 + 7,80) В = 12,0 В.

Стратегия и решение для (d)

Самый простой способ рассчитать мощность в ваттах (Вт), рассеиваемую резистором в цепи постоянного тока, — это использовать закон Джоуля , P = IV , где P — электрическая мощность. В этом случае через каждый резистор протекает одинаковый полный ток.Подставляя закон Ома V = IR в закон Джоуля, мы получаем мощность, рассеиваемую первым резистором, как

P 1 = I 2 R 1 = (0,600 A) 2 (1,00 Ом) = 0,360 Вт

Аналогично

P 2 = I 2 R 2 = (0,600 A) 2 (6,00 Ом) = 2,16 Вт

и

P 3 = I 2 R 3 = (0.{2}} {R} \\ [/ latex], где В, — это падение напряжения на резисторе (а не полное напряжение источника). Получатся те же значения.

Стратегия и решение для (e)

Самый простой способ рассчитать выходную мощность источника — использовать P = IV , где В, — напряжение источника. Это дает

P = (0,600 A) (12,0 В) = 7,20 Вт.

Обсуждение для (e)

Обратите внимание, что по совпадению общая мощность, рассеиваемая резисторами, также равна 7.20 Вт, столько же, сколько мощность, выдаваемая источником. То есть

P 1 + P 2 + P 3 = (0,360 + 2,16 + 4,68) W = 7,20 Вт.

Мощность — это энергия в единицу времени (ватт), поэтому для сохранения энергии требуется, чтобы выходная мощность источника была равна общей мощности, рассеиваемой резисторами.

Основные характеристики резисторов серии

  1. Последовательные сопротивления добавить: R с = R 1 + R 2 + R 3 +….
  2. Одинаковый ток протекает последовательно через каждый резистор.
  3. Отдельные последовательно включенные резисторы не получают полное напряжение источника, а делят его.

На рисунке 3 показаны резисторы , включенные параллельно , подключенные к источнику напряжения. Резисторы включены параллельно, когда каждый резистор подключен непосредственно к источнику напряжения с помощью соединительных проводов с незначительным сопротивлением. Таким образом, к каждому резистору приложено полное напряжение источника. Каждый резистор потребляет такой же ток, как если бы он один был подключен к источнику напряжения (при условии, что источник напряжения не перегружен).Например, автомобильные фары, радио и т. Д. Подключены параллельно, так что они используют полное напряжение источника и могут работать полностью независимо. То же самое и в вашем доме, или в любом другом здании. (См. Рисунок 3 (b).)

Рис. 3. (a) Три резистора, подключенных параллельно батарее, и эквивалентное одиночное или параллельное сопротивление. (б) Электроснабжение в доме. (Источник: Dmitry G, Wikimedia Commons)

Чтобы найти выражение для эквивалентного параллельного сопротивления R p , давайте рассмотрим протекающие токи и их связь с сопротивлением.Поскольку каждый резистор в цепи имеет полное напряжение, токи, протекающие через отдельные резисторы, равны [латекс] {I} _ {1} = \ frac {V} {{R} _ {1}} \\ [/ latex] , [латекс] {I} _ {2} = \ frac {V} {{R} _ {2}} \\ [/ latex] и [латекс] {I} _ {3} = \ frac {V} {{R} _ {3}} \\ [/ латекс]. Сохранение заряда подразумевает, что полный ток I , производимый источником, является суммой этих токов:

I = I 1 + I 2 + I 3 .

Подстановка выражений для отдельных токов дает

[латекс] I = \ frac {V} {{R} _ {1}} + \ frac {V} {{R} _ {2}} + \ frac {V} {{R} _ {3}} = V \ left (\ frac {1} {{R} _ {1}} + \ frac {1} {{R} _ {2}} + \ frac {1} {{R} _ {3}} \ справа) \\ [/ латекс].

Обратите внимание, что закон Ома для эквивалентного одиночного сопротивления дает

[латекс] I = \ frac {V} {{R} _ {p}} = V \ left (\ frac {1} {{R} _ {p}} \ right) \\ [/ latex].

Члены в круглых скобках в последних двух уравнениях должны быть равны. Обобщая для любого количества резисторов, общее сопротивление R p параллельного соединения связано с отдельными сопротивлениями на

[латекс] \ frac {1} {{R} _ {p}} = \ frac {1} {{R} _ {1}} + \ frac {1} {{R} _ {2}} + \ гидроразрыв {1} {{R} _ {\ text {.} 3}} + \ text {.} \ Text {…} \\ [/ latex]

Это соотношение приводит к общему сопротивлению R p , которое меньше наименьшего из отдельных сопротивлений. (Это видно в следующем примере.) При параллельном подключении резисторов от источника течет больше тока, чем протекает по любому из них по отдельности, поэтому общее сопротивление ниже.

Пример 2. Расчет сопротивления, тока, рассеиваемой мощности и выходной мощности: анализ параллельной цепи

Пусть выходное напряжение батареи и сопротивления в параллельном соединении на Рисунке 3 будут такими же, как в ранее рассмотренном последовательном соединении: В = 12.0 В, R 1 = 1,00 Ом, R 2 = 6,00 Ом и R 3 = 13,0 Ом. а) Каково полное сопротивление? (б) Найдите полный ток. (c) Рассчитайте токи в каждом резисторе и покажите, как они складываются, чтобы равняться общему выходному току источника. (d) Рассчитайте мощность, рассеиваемую каждым резистором. (e) Найдите выходную мощность источника и покажите, что она равна общей мощности, рассеиваемой резисторами.

Стратегия и решение для (а)

Общее сопротивление для параллельной комбинации резисторов находится с помощью следующего уравнения.Ввод известных значений дает

[латекс] \ frac {1} {{R} _ {p}} = \ frac {1} {{R} _ {1}} + \ frac {1} {{R} _ {2}} + \ frac {1} {{R} _ {3}} = \ frac {1} {1 \ text {.} \ text {00} \ text {} \ Omega} + \ frac {1} {6 \ text {. } \ text {00} \ text {} \ Omega} + \ frac {1} {\ text {13} \ text {.} 0 \ text {} \ Omega} \\ [/ latex].

Таким образом,

[латекс] \ frac {1} {{R} _ {p}} = \ frac {1.00} {\ text {} \ Omega} + \ frac {0 \ text {.} \ Text {1667}} {\ текст {} \ Omega} + \ frac {0 \ text {.} \ text {07692}} {\ text {} \ Omega} = \ frac {1 \ text {.} \ text {2436}} {\ text { } \ Omega} \\ [/ латекс].

(Обратите внимание, что в этих расчетах каждый промежуточный ответ отображается с дополнительной цифрой.) Мы должны перевернуть это, чтобы найти полное сопротивление R p . Это дает

[латекс] {R} _ {\ text {p}} = \ frac {1} {1 \ text {.} \ Text {2436}} \ text {} \ Omega = 0 \ text {.} \ Text { 8041} \ text {} \ Omega \\ [/ latex].

Общее сопротивление с правильным количеством значащих цифр составляет R p = 0,804 Ом

Обсуждение для (а)

R p , как и предполагалось, меньше наименьшего индивидуального сопротивления.

Стратегия и решение для (b)

Полный ток можно найти из закона Ома, заменив полное сопротивление R p . Это дает

[латекс] I = \ frac {V} {{R} _ {\ text {p}}} = \ frac {\ text {12.0 V}} {0.8041 \ text {} \ Omega} = \ text {14} \ text {.} \ text {92 A} \\ [/ latex].

Обсуждение для (б)

Ток I для каждого устройства намного больше, чем для тех же устройств, подключенных последовательно (см. Предыдущий пример).Схема с параллельным соединением имеет меньшее общее сопротивление, чем резисторы, включенные последовательно.

Стратегия и решение для (c)

Отдельные токи легко вычислить по закону Ома, поскольку каждый резистор получает полное напряжение. Таким образом,

[латекс] {I} _ {1} = \ frac {V} {{R} _ {1}} = \ frac {12.0 \ text {V}} {1.00 \ text {} \ Omega} = 12.0 \ text {A} \\ [/ латекс].

Аналогично

[латекс] {I} _ {2} = \ frac {V} {{R} _ {2}} = \ frac {12.0 \ text {V}} {6.00 \ text {} \ Omega} = 2 \ text {.} \ text {00} \ text {A} \\ [/ latex]

и

[латекс] {I} _ {3} = \ frac {V} {{R} _ {3}} = \ frac {\ text {12} \ text {.} 0 \ text {V}} {\ text {13} \ text {.} \ Text {0} \ text {} \ Omega} = 0 \ text {.} \ Text {92} \ text {A} \\ [/ latex].

Обсуждение для (c)

Общий ток складывается из отдельных токов:

I 1 + I 2 + I 3 = 14,92 A.

Это соответствует сохранению заряда.{2}} {13.0 \ text {} \ Omega} = 11.1 \ text {W} \\ [/ latex].

Обсуждение для (d)

Мощность, рассеиваемая каждым резистором при параллельном подключении, значительно выше, чем при последовательном подключении к тому же источнику напряжения.

Стратегия и решение для (e)

Общую мощность также можно рассчитать несколькими способами. Выбрав P = IV и введя полный ток, получим

P = IV = (14,92 A) (12,0 В) = 179 Вт.

Обсуждение для (e)

Общая мощность, рассеиваемая резисторами, также составляет 179 Вт:

P 1 + P 2 + P 3 = 144 Вт + 24,0 Вт + 11,1 Вт = 179 Вт

Это соответствует закону сохранения энергии.

Общее обсуждение

Обратите внимание, что как токи, так и мощность при параллельном подключении больше, чем для тех же устройств, подключенных последовательно.

Основные характеристики резисторов, подключенных параллельно
  1. Параллельное сопротивление определяется из [latex] \ frac {1} {{R} _ {\ text {p}}} = \ frac {1} {{R} _ {1}} + \ frac {1} { {R} _ {2}} + \ frac {1} {{R} _ {3}} + \ text {…} \\ [/ latex], и оно меньше любого отдельного сопротивления в комбинации.
  2. На каждый параллельно включенный резистор подается такое же полное напряжение источника. (В системах распределения электроэнергии чаще всего используются параллельные соединения для питания бесчисленных устройств, обслуживаемых одним и тем же напряжением, и для того, чтобы они могли работать независимо.)
  3. Параллельные резисторы не получают суммарный ток каждый; они делят это.

Сочетания последовательного и параллельного

Более сложные соединения резисторов иногда представляют собой просто комбинации последовательного и параллельного. Они обычно встречаются, особенно если учитывать сопротивление провода. В этом случае сопротивление провода включено последовательно с другими сопротивлениями, включенными параллельно. Комбинации последовательного и параллельного подключения можно свести к одному эквивалентному сопротивлению, используя метод, показанный на рисунке 4.Различные части идентифицируются как последовательные или параллельные, уменьшаются до их эквивалентов и далее уменьшаются до тех пор, пока не останется единственное сопротивление. Процесс занимает больше времени, чем труден.

Рис. 4. Эта комбинация из семи резисторов имеет как последовательные, так и параллельные части. Каждое из них идентифицируется и приводится к эквивалентному сопротивлению, а затем уменьшается до тех пор, пока не будет достигнуто одно эквивалентное сопротивление.

Самая простая комбинация последовательного и параллельного сопротивления, показанная на рисунке 4, также является наиболее поучительной, поскольку она используется во многих приложениях.Например, R 1 может быть сопротивлением проводов от автомобильного аккумулятора к его электрическим устройствам, которые подключены параллельно. R 2 и R 3 могли быть стартером и светом салона. Ранее мы предполагали, что сопротивление провода незначительно, но, когда это не так, оно имеет важные последствия, как показывает следующий пример.

Пример 3. Расчет сопротивления,

IR Падение, ток и рассеиваемая мощность: объединение последовательных и параллельных цепей

На рис. 5 показаны резисторы из двух предыдущих примеров, подключенные другим способом — сочетание последовательного и параллельного.Можно считать R 1 сопротивлением проводов, ведущих к R 2 и R 3 . (а) Найдите полное сопротивление. (b) Что такое падение IR в R 1 ? (c) Найдите текущие значения от I 2 до R 2 . (d) Какую мощность рассеивает R 2 ?

Рис. 5. Эти три резистора подключены к источнику напряжения, так что R 2 и R 3 параллельны друг другу, и эта комбинация включена последовательно с R 1 .

Стратегия и решение для (а)

Чтобы найти полное сопротивление, отметим, что R 2 и R 3 находятся параллельно, и их комбинация R p последовательно с R 1 . Таким образом, полное (эквивалентное) сопротивление этой комбинации составляет

.

R итого = R 1 + R p .

Сначала мы находим R p , используя уравнение для параллельных резисторов и вводя известные значения:

[латекс] \ frac {1} {{R} _ {\ text {p}}} = \ frac {1} {{R} _ {2}} + \ frac {1} {{R} _ {3 }} = \ frac {1} {6 \ text {.} \ text {00} \ text {} \ Omega} + \ frac {1} {\ text {13} \ text {.} 0 \ text {} \ Omega} = \ frac {0.2436} {\ text {} \ Омега} \\ [/ латекс].

Инвертирование дает

[латекс] {R} _ {\ text {p}} = \ frac {1} {0,2436} \ text {} \ Omega = 4.11 \ text {} \ Omega \\ [/ latex].

Таким образом, общее сопротивление равно

.

R до = R 1 + R p = 1,00 Ом + 4,11 Ом = 5,11 Ом.

Обсуждение для (а)

Общее сопротивление этой комбинации является промежуточным между значениями чистой серии и чистой параллели (20.0 Ом и 0,804 Ом соответственно), найденные для тех же резисторов в двух предыдущих примерах.

Стратегия и решение для (b)

Чтобы найти падение IR в R 1 , отметим, что полный ток I протекает через R 1 . Таким образом, его падение IR составляет

В 1 = ИК 1

Мы должны найти I , прежде чем сможем вычислить V 1 .Полный ток I находится с помощью закона Ома для схемы. То есть

[латекс] I = \ frac {V} {{R} _ {\ text {tot}}} = \ frac {\ text {12.0} \ text {V}} {5.11 \ text {} \ Omega} = 2.35 \ text {A} \\ [/ latex].

Вводя это в выражение выше, мы получаем

В 1 = IR 1 = (2,35 A) (1,00 Ом) = 2,35 В.

Обсуждение для (б)

Напряжение, приложенное к R 2 и R 3 , меньше полного напряжения на величину В 1 .Когда сопротивление провода велико, это может существенно повлиять на работу устройств, представленных R 2 и R 3 .

Стратегия и решение для (c)

Чтобы найти ток через R 2 , мы должны сначала найти приложенное к нему напряжение. Мы называем это напряжение В p , потому что оно приложено к параллельной комбинации резисторов. Напряжение, приложенное как к R 2 , так и к R 3 , уменьшается на величину В 1 , и поэтому оно составляет

В p = V V 1 = 12.0 В — 2,35 В = 9,65 В.

Теперь ток I 2 через сопротивление R 2 находится по закону Ома:

[латекс] {I} _ {2} = \ frac {{V} _ {\ text {p}}} {{R} _ {2}} = \ frac {9.65 \ text {V}} {6.00 \ текст {} \ Omega} = 1,61 \ text {A} \\ [/ latex].

Обсуждение для (c)

Ток меньше 2,00 А, которые протекали через R 2 , когда он был подключен параллельно к батарее в предыдущем примере параллельной цепи.

Стратегия и решение для (d)

Мощность, рассеиваемая R 2 равна

P 2 = ( I 2 ) 2 R 2 = (1,61 A) 2 (6,00 Ом) = 15,5 Вт

Обсуждение для (d)

Мощность меньше 24,0 Вт, рассеиваемых этим резистором при параллельном подключении к источнику 12,0 В.

Одним из следствий этого последнего примера является то, что сопротивление в проводах снижает ток и мощность, подаваемую на резистор.Если сопротивление провода относительно велико, как в изношенном (или очень длинном) удлинителе, то эти потери могут быть значительными. Если потребляется большой ток, падение IR в проводах также может быть значительным.

Например, когда вы роетесь в холодильнике и включается мотор, свет холодильника на мгновение гаснет. Точно так же вы можете увидеть тусклый свет в салоне, когда вы запускаете двигатель вашего автомобиля (хотя это может быть связано с сопротивлением внутри самой батареи).

То, что происходит в этих сильноточных ситуациях, показано на рисунке 6. Устройство, обозначенное номером R 3 , имеет очень низкое сопротивление, поэтому при его включении протекает большой ток. Этот увеличенный ток вызывает большее падение IR в проводах, представленных R 1 , уменьшая напряжение на лампочке (которое составляет R 2 ), которое затем заметно гаснет.

Рис. 6. Почему гаснет свет при включении большого прибора? Ответ заключается в том, что большой ток, потребляемый двигателем прибора, вызывает значительное падение напряжения в проводах и снижает напряжение на свету.

Проверьте свое понимание

Можно ли любую произвольную комбинацию резисторов разбить на последовательную и параллельную? Посмотрите, сможете ли вы нарисовать принципиальную схему резисторов, которые нельзя разбить на комбинации последовательно и параллельно.

Решение Нет, есть много способов подключения резисторов, которые не являются комбинациями последовательного и параллельного, включая петли и переходы. В таких случаях правила Кирхгофа, которые будут включены в Правила Кирхгофа, позволят вам проанализировать схему.

Стратегии решения проблем для последовательных и параллельных резисторов
  1. Нарисуйте четкую принципиальную схему, пометив все резисторы и источники напряжения. Этот шаг включает список известных проблем, поскольку они отмечены на вашей принципиальной схеме.
  2. Определите, что именно необходимо определить в проблеме (определите неизвестные). Письменный список полезен.
  3. Определите, включены ли резисторы последовательно, параллельно или в комбинации последовательно и параллельно.Изучите принципиальную схему, чтобы сделать эту оценку. Резисторы включены последовательно, если через них должен последовательно проходить один и тот же ток.
  4. Используйте соответствующий список основных функций для последовательных или параллельных подключений, чтобы найти неизвестные. Есть один список для серий, а другой — для параллелей. Если ваша проблема представляет собой комбинацию последовательного и параллельного соединения, уменьшайте ее поэтапно, рассматривая отдельные группы последовательных или параллельных соединений, как это сделано в этом модуле и примерах. Особое примечание: при нахождении R необходимо соблюдать осторожность.
  5. Проверьте, являются ли ответы разумными и последовательными. Единицы и числовые результаты должны быть разумными. Общее последовательное сопротивление должно быть больше, а общее параллельное сопротивление, например, должно быть меньше. Мощность должна быть больше для одних и тех же устройств, подключенных параллельно, по сравнению с последовательными и так далее.

Сводка раздела

Концептуальные вопросы

1. Переключатель имеет переменное сопротивление, близкое к нулю в замкнутом состоянии и очень большое в разомкнутом, и он включен последовательно с устройством, которым он управляет.Объясните влияние переключателя на рис. 7 на ток в разомкнутом и замкнутом состоянии.

Рис. 7. Переключатель обычно включается последовательно с источником сопротивления и напряжения. В идеале переключатель имеет почти нулевое сопротивление в замкнутом состоянии, но имеет чрезвычайно большое сопротивление в разомкнутом состоянии. (Обратите внимание, что на этой диаграмме скрипт E представляет напряжение (или электродвижущую силу) батареи.)

2. Какое напряжение на разомкнутом переключателе на Рисунке 7?

3. На разомкнутом переключателе есть напряжение, как на Рисунке 7.Почему же тогда мощность, рассеиваемая разомкнутым переключателем, мала?

4. Почему мощность, рассеиваемая замкнутым переключателем, как на Рисунке 7, мала?

5. Студент в физической лаборатории по ошибке подключил электрическую лампочку, батарею и выключатель, как показано на рисунке 8. Объясните, почему лампочка горит, когда выключатель разомкнут, и гаснет, когда он замкнут. (Не пытайтесь — батарея сильно разряжается!)

Рис. 8. Ошибка подключения. Включите этот переключатель параллельно устройству, обозначенному [латекс] R [/ латекс].(Обратите внимание, что на этой диаграмме скрипт E представляет напряжение (или электродвижущую силу) батареи.)

6. Зная, что сила электрического шока зависит от величины тока, протекающего через ваше тело, вы бы предпочли, чтобы он был включен последовательно или параллельно с сопротивлением, таким как нагревательный элемент тостера, если он шокирован им? Объяснять.

7. Были бы ваши фары тусклыми при запуске двигателя автомобиля, если бы провода в вашем автомобиле были сверхпроводниками? (Не пренебрегайте внутренним сопротивлением батареи.) Объяснять.

8. Некоторые гирлянды праздничных огней соединены последовательно для экономии затрат на проводку. В старой версии использовались лампочки, которые при перегорании разрывают электрическое соединение, как выключатель. Если одна такая лампочка перегорит, что случится с остальными? Если такая цепочка работает от 120 В и имеет 40 одинаковых лампочек, каково нормальное рабочее напряжение каждой? В более новых версиях используются лампы, которые при перегорании замыкаются накоротко, как замкнутый выключатель. Если одна такая лампочка перегорит, что случится с остальными? Если такая цепочка работает от 120 В и в ней осталось 39 идентичных лампочек, каково тогда рабочее напряжение каждой?

9.Если две бытовые лампочки мощностью 60 Вт и 100 Вт подключить последовательно к бытовой электросети, какая из них будет ярче? Объяснять.

10. Предположим, вы проводите физическую лабораторию, в которой вас просят вставить резистор в цепь, но все прилагаемые резисторы имеют большее сопротивление, чем запрошенное значение. Как бы вы соединили доступные сопротивления, чтобы попытаться получить меньшее запрошенное значение?

11. Перед Второй мировой войной некоторые радиостанции получали питание через «шнур сопротивления», который имел значительное сопротивление.Такой резистивный шнур снижает напряжение до желаемого уровня для ламп радиоприемника и т.п., и это экономит расходы на трансформатор. Объясните, почему шнуры сопротивления нагреваются и тратят энергию при включенном радио.

12. У некоторых лампочек есть три уровня мощности (не включая ноль), получаемые от нескольких нитей накала, которые индивидуально переключаются и соединяются параллельно. Какое минимальное количество нитей нити необходимо для трех режимов мощности?

Задачи и упражнения

Примечание. Можно считать, что данные, взятые из цифр, имеют точность до трех значащих цифр.

1. (а) Каково сопротивление десяти последовательно соединенных резисторов сопротивлением 275 Ом? (б) Параллельно?

2. (a) Каково сопротивление последовательно соединенных резисторов 1,00 × 10 2 Ом, 2,50 кОм и 4,00 кОм? (б) Параллельно?

3. Какое наибольшее и наименьшее сопротивление можно получить, соединив резисторы на 36,0 Ом, 50,0 Ом и 700 Ом?

4. Тостер на 1800 Вт, электрическая сковорода на 1400 Вт и лампа на 75 Вт подключены к одной розетке в цепи 15 А, 120 В.(Три устройства работают параллельно, если подключены к одной розетке.) а) Какой ток потребляет каждое устройство? (b) Перегорит ли эта комбинация предохранитель на 15 А?

5. Фара мощностью 30,0 Вт и стартер мощностью 2,40 кВт обычно подключаются параллельно в систему на 12,0 В. Какую мощность потребляли бы одна фара и стартер при последовательном подключении к батарее 12,0 В? (Не обращайте внимания на любое другое сопротивление в цепи и любое изменение сопротивления в двух устройствах.)

6.(a) Для батареи 48,0 В и резисторов 24,0 Ом и 96,0 Ом найдите для каждого из них ток и мощность при последовательном соединении. (b) Повторите, когда сопротивления включены параллельно.

7. Ссылаясь на пример комбинирования последовательных и параллельных цепей и рисунок 5, вычислите I 3 двумя следующими способами: (a) по известным значениям I и I 2 ; (б) используя закон Ома для R 3 . В обеих частях явно показано, как вы следуете шагам, описанным выше в стратегии решения проблем для последовательных и параллельных резисторов .

Рис. 5. Эти три резистора подключены к источнику напряжения, так что R 2 и R 3 параллельны друг другу, и эта комбинация включена последовательно с R 1 .

8. Ссылаясь на рисунок 5: (a) Вычислите P 3 и обратите внимание на его сравнение с P 3 , найденным в первых двух примерах задач в этом модуле. (b) Найдите полную мощность, отдаваемую источником, и сравните ее с суммой мощностей, рассеиваемых резисторами.

9. См. Рисунок 6 и обсуждение затемнения света при включении тяжелого прибора. (а) Учитывая, что источник напряжения составляет 120 В, сопротивление провода составляет 0,400 Ом, а номинальная мощность лампы составляет 75,0 Вт, какая мощность будет рассеиваться лампой, если при включении двигателя через провода пройдет в общей сложности 15,0 А? Предположите незначительное изменение сопротивления лампы. б) Какая мощность потребляет двигатель?

Рис. 6. Почему гаснет свет при включении большого прибора? Ответ заключается в том, что большой ток, потребляемый двигателем прибора, вызывает значительное падение напряжения в проводах и снижает напряжение на свету.

10. Линия электропередачи 240 кВ с 5,00 × 10 2 подвешена к заземленным металлическим опорам с помощью керамических изоляторов, каждый из которых имеет сопротивление 1,00 × 10 9 Ом (рисунок 9 (a)). Какое сопротивление на землю у 100 изоляторов? (b) Рассчитайте мощность, рассеиваемую 100 из них. (c) Какая доля мощности, переносимой линией, составляет это? Ясно покажите, как вы следуете шагам, описанным выше в стратегии решения проблем для последовательных и параллельных резисторов .

Рис. 9. Высоковольтная (240 кВ) линия электропередачи 5,00 × 10 2 подвешена к заземленной металлической опоре электропередачи. Ряд керамических изоляторов обеспечивает сопротивление 1,00 × 10 9 Ом каждый.

11. Покажите, что если два резистора R 1 и R 2 объединены, и один из них намного больше другого ( R 1 >> R 2 ): (a ) Их последовательное сопротивление почти равно большему сопротивлению R 1 .(б) Их параллельное сопротивление почти равно меньшему сопротивлению R 2 .

12. Необоснованные результаты Два резистора, один из которых имеет сопротивление 145 Ом, подключены параллельно, чтобы получить общее сопротивление 150 Ом. а) Каково значение второго сопротивления? б) Что неразумного в этом результате? (c) Какие предположения необоснованны или непоследовательны?

13. Необоснованные результаты Два резистора, один из которых имеет сопротивление 900 кОм, соединены последовательно, чтобы получить общее сопротивление 0.500 МОм. а) Каково значение второго сопротивления? б) Что неразумного в этом результате? (c) Какие предположения необоснованны или непоследовательны?

Глоссарий

серия:
последовательность резисторов или других компонентов, включенных в цепь один за другим
резистор:
компонент, обеспечивающий сопротивление току, протекающему через электрическую цепь
сопротивление:
вызывает потерю электроэнергии в цепи
Закон Ома:
соотношение между током, напряжением и сопротивлением в электрической цепи: В = IR
напряжение:
электрическая потенциальная энергия на единицу заряда; электрическое давление, создаваемое источником питания, например аккумулятором
падение напряжения:
потеря электроэнергии при прохождении тока через резистор, провод или другой компонент
ток:
поток заряда через электрическую цепь мимо заданной точки измерения
Закон Джоуля:
взаимосвязь между потенциальной электрической мощностью, напряжением и сопротивлением в электрической цепи, определяемая следующим образом: [latex] {P} _ {e} = \ text {IV} [/ latex]
параллельно:
разводку резисторов или других компонентов в электрической цепи, так что каждый компонент получает одинаковое напряжение от источника питания; часто изображается на диаграмме в виде лестницы, где каждый компонент находится на ступеньке лестницы

Избранные решения проблем и упражнения

1.(а) 2,75 кОм (б) 27,5 Ом

3. (а) 786 Ом (б) 20,3 Ом

5. 29,6 Вт

7. (а) 0,74 А (б) 0,742 А

9. (а) 60,8 Вт (б) 3,18 кВт

11. (a) [латекс] \ begin {array} {} {R} _ {\ text {s}} = {R} _ {1} + {R} _ {2} \\ \ Rightarrow {R} _ {\ text {s}} \ приблизительно {R} _ {1} \ left ({R} _ {1} \ text {>>} {R} _ {2} \ right) \ end {array} \\ [/ латекс]

(b) [латекс] \ frac {1} {{R} _ {p}} = \ frac {1} {{R} _ {1}} + \ frac {1} {{R} _ {2} } = \ frac {{R} _ {1} + {R} _ {2}} {{R} _ {1} {R} _ {2}} \\ [/ latex],

так что

[латекс] \ begin {array} {} {R} _ {p} = \ frac {{R} _ {1} {R} _ {2}} {{R} _ {1} + {R} _ {2}} \ приблизительно \ frac {{R} _ {1} {R} _ {2}} {{R} _ {1}} = {R} _ {2} \ left ({R} _ {1 } \ text {>>} {R} _ {2} \ right) \ text {.} \ end {array} \\ [/ latex]

13. (a) –400 кОм (b) Сопротивление не может быть отрицательным. (c) Считается, что последовательное сопротивление меньше, чем у одного из резисторов, но должно быть больше, чем у любого из резисторов.

резисторов параллельно | Прядильные числа

Компоненты находятся в параллельных , если они совместно используют два узла, например,

Мы раскрываем свойства параллельного соединения, рассматривая схемы резисторов. Определить параллельные резисторы немного сложнее, чем последовательные резисторы.

Автор Вилли Макаллистер.


Содержание


Параллельное соединение

Компоненты работают параллельно, когда их два терминала подключены к одним и тем же узлам.

На следующем изображении $ \ text R1 $, $ \ text R2 $ и $ \ text R3 $ расположены параллельно. Две горизонтальные линии представляют два узла, общие для всех трех резисторов.

Просмотрите определение узла

Просмотрите определения узла и распределенного узла в этой статье о терминологии схемы.Еще один хороший ресурс, который поможет вам разобраться в параллельных схемах, — это статья об эквивалентности схем.

Поскольку резисторы подключены к одним и тем же узлам, на каждом параллельном резисторе появляется одинаковое напряжение.

Сопротивления на следующем изображении — это , а не , подключенные параллельно. Есть дополнительные компоненты (оранжевые прямоугольники), разделяющие общие узлы между резисторами. Эта схема имеет четыре отдельных узла, поэтому $ \ text R1 $, $ \ text R2 $ и $ \ text R3 $ не имеют одного и того же напряжения.

Резисторы параллельно

Вот схема с параллельными резисторами. В этой схеме есть источник тока. Мы не используем их очень часто, так что это должно быть весело.

Текущий источник $ \ text I_ \ text s $ направляет текущий $ i $ к $ \ text R1 $, $ \ text R2 $ и $ \ text R3 $. Текущий $ i $ возвращается на нижний терминал текущего источника. но мы еще не знаем, какое напряжение $ v $ или как $ i $ распадается на токи трех резисторов.

То, что мы знаем,

  • Текущее значение $ i $ — некоторое заданное постоянное значение.
  • Сумма токов трех резисторов должна составлять $ i $.
  • Напряжение $ v $ появляется на всех трех резисторах.

С помощью всего лишь этого небольшого количества знаний и закона Ома мы можем найти то, чего не знаем,

  • Нам пока неизвестно напряжение $ v $, появляющееся на резисторах и источнике тока.
  • Мы не знаем, как $ i $ распадается на токи трех резисторов.

Используя то, что мы знаем, мы можем записать эти выражения,

$ i = i _ {\ text R1} + i _ {\ text R2} + i _ {\ text R3}

$

$ v = i _ {\ text R1} \, \ text R1 \ qquad v = i _ {\ text R2} \, \ text R2 \ qquad v = i _ {\ text R3} \, \ text R3 $

Переставьте три выражения закона Ома, чтобы найти ток,

$ i _ {\ text R1} = \ dfrac {v} {\ text R1} \ qquad i _ {\ text R2} = \ dfrac {v} {\ text R2} \ qquad i _ {\ text R3} = \ dfrac { v} {\ text R3}

долларов США

Подставьте эти выражения в сумму токов,

$ i = \ dfrac {v} {\ text R1} + \ dfrac {v} {\ text R2} + \ dfrac {v} {\ text R3}

$

Выносим за скобки общий термин $ v $,

$ i = v \ left (\ dfrac {1} {\ text R1} + \ dfrac {1} {\ text R2} + \ dfrac {1} {\ text R3} \ right) $

Помните, что мы уже знаем $ i $ (он установлен в текущем источнике), поэтому мы можем решить для $ v $,

$ v = i \, \ dfrac {1} {\ left (\ dfrac {1} {\ text R1} + \ dfrac {1} {\ text R2} + \ dfrac {1} {\ text R3} \ right )}

долларов

Взгляните на это уравнение.Это похоже на закон Ома, $ v = i \, \ text R $, но с резисторами, представленными в виде большого двойного обратного выражения, где обычно появляется $ \ text R $.

Заключаем,

Для резисторов, включенных параллельно, общее сопротивление обратно пропорционально сумме обратных величин отдельных резисторов.

(Это полнота. Легче понять уравнение, чем произносить его вслух.)

Эквивалентный параллельный резистор

Предыдущее уравнение предлагает способ определения нового резистора , эквивалентного трем параллельным резисторам.

$ \ text R _ {\ text {parallel}} = \ dfrac {1} {\ left (\ dfrac {1} {\ text R1} + \ dfrac {1} {\ text R2} + \ dfrac {1} { \ text R3} \ right)}

долларов США

Новый резистор является величиной, обратной сумме обратных величин. Это эквивалентно в том смысле, что для данного $ i $, вытекающего из источника тока, появляется такое же напряжение $ v $.

Мы можем записать это немного меньше, перевернув гигантское число, равное

.

$ \ dfrac {1} {\ text R _ {\ text {parallel}}} = \ dfrac {1} {\ text R1} + \ dfrac {1} {\ text R2} + \ dfrac {1} {\ text R3}

долларов

Вы можете заменить три параллельных резистора одним эквивалентным резистором, и $ i $ и $ v $ останутся неизменными.

С «точки зрения» источника тока вы не сможете отличить эквивалентный резистор от трех параллельных резисторов.

$ \ text N $ резисторы параллельно

Мы можем обобщить уравнение параллельного резистора для любого количества резисторов, включенных параллельно. Вывод этой формулы для резисторов $ \ text N $, включенных параллельно, следует точно таким же шагам, которые мы проделали выше для трех резисторов, включенных параллельно.

$ \ dfrac {1} {\ text R _ {\ text {parallel}}} = \ dfrac {1} {\ text R1} + \ dfrac {1} {\ text R2} + \ ldots + \ dfrac {1} {\ text {R} _ \ text N}

долларов США

$ \ qquad \ quad $

Ток распределяется между резисторами, включенными параллельно

Давайте выясним, как основной ток $ i $ разделяется и течет через каждый резистор.Мы ищем $ i _ {\ text R1} $, $ i _ {\ text R2} $ и $ i _ {\ text R3} $.

Примените закон Ома к отдельным резисторам. $ v $ одинаково во всех уравнениях,

$ i _ {\ text R1} = v / \ text R_1 \ qquad i _ {\ text R2} = v / \ text R_2 \ qquad i _ {\ text R3} = v / \ text R_3 $

Основной ток $ i $ разделяется на три отдельных тока. Эти токи воссоединяются и текут обратно к источнику тока. Это означает, что токи отдельных резисторов должны суммироваться с основным током.

Величина тока в каждом резисторе составляет обратно пропорционально номиналу резистора.

Это будет лучше, если мы сделаем пример с действительными числами.

Проверка концепции

ПРОБЛЕМА 1

а. Найдите напряжение $ v $.

г. Найдите токи в трех резисторах.

г. Покажите, как в сумме токи отдельных резисторов составляют $ i $.

Шаги к решению:

  1. Найдите эквивалентное параллельное сопротивление $ \ text R _ {\ text {parallel}} $.
  2. Найдите напряжение $ v $, используя закон Ома.
  3. Найдите отдельные токи, снова используя закон Ома.
  4. Убедитесь, что токи резистора в сумме соответствуют требуемым.
Показать ответ

Часть а. Эквивалент $ \ text R _ {\ text {parallel}} $ равен обратной обратной сумме трех резисторов.

$ \ text R _ {\ text {parallel}} = \ dfrac {1} {\ left (\ dfrac {1} {50 \, \ Omega} + \ dfrac {1} {100 \, \ Omega} + \ dfrac {1} {500 \, \ Omega} \ right)}

долларов США

$ \ text R _ {\ text {parallel}} = \ dfrac {1} {(0.02 + 0,01 + 0,002)} = \ dfrac {1} {0,032} = 31,25 \, \ Omega $

Теперь мы знаем ток и эквивалентное сопротивление. Мы можем найти напряжение $ v $ на всех резисторах, используя закон Ома

.

$ v = i \, \ text R _ {\ text {parallel}}

$

$ v = 100 \, \ text {mA} \ cdot 31.25 \, \ Omega = 3.125 \, \ text V $

Часть б. Зная $ v $, мы можем вернуться и вычислить токи отдельных резисторов,

$ i _ {\ text R1} = \ dfrac {v} {\ text R1} = \ dfrac {3.125 \, \ text V} {\ text {50}} = 62,5 \, \ text {mA}

долларов США

$ i _ {\ text R2} = \ dfrac {v} {\ text R2} = \ dfrac {3.125 \, \ text V} {\ text {100}} = 31.25 \, \ text {mA}

$

$ i _ {\ text R3} = \ dfrac {v} {\ text R3} = \ dfrac {3.125 \, \ text V} {\ text {500}} = \ phantom {0} 6.25 \, \ text {mA } $

Вот как выглядит решение,

Часть c. Проверка: Суммируются ли токи резистора с током источника?

$ 62.5 \, \ text {mA} + 31.25 \, \ text {mA} + 6.25 \, \ text {mA} = 100 \, \ text {mA} \ qquad \ checkmark $ Да!

Как электроны «знают», через какой резистор пройти?

Распространенный вопрос о параллельных цепях: «Когда ток достигает первого резистора, как электроны« выбирают »или« знают », через какой резистор протекать?» Или «Как электроны« решают », куда идти? ”

Как известно, электроны — не люди, у них нет маленьких электронных мозгов (насколько нам известно).Здесь нет выбора или принятия решения. Они не выбирают и не решают, через какой резистор протекать, точно так же, как молекула воды «решает» или «выбирает», через какую сторону камня она будет проходить в потоке. В любой цепи, включая параллельные цепи, каждый электрон реагирует только на электрические силы от напряжения и от отталкивающего роя окружающих электронов.

На основе только что найденных токов резисторов,

ПРОБЛЕМА 2

Через какой резистор проходит наибольшая доля тока?

Выберите один: $ \ quad $ наименьший $ \ quad $ средний $ \ quad $ наибольший

покажи ответ

Самый маленький резистор несет наибольшую долю тока.

ПРОБЛЕМА 3

Через какой резистор проходит наименьшая доля тока?

Выберите один: $ \ quad $ наименьший $ \ quad $ средний $ \ quad $ наибольший

покажи ответ

Самый большой резистор проводит наименьшую долю тока.

ПРОБЛЕМА 4

Каков относительный размер эквивалентного резистора по сравнению с параллельными резисторами?

  • $ \ text {R} _ \ text {parallel} $ больше, чем самый большой резистор.
  • $ \ text {R} _ \ text {parallel} $ находится между самым большим и самым маленьким резистором.
  • $ \ text {R} _ \ text {parallel} $ меньше самого маленького резистора.
показать ответ

Эквивалентный параллельный резистор меньше самого маленького параллельного резистора. Всегда.

Проблема 5

Какой резистор имеет наибольшее напряжение?

Выберите один: $ \ quad $ наименьший $ \ quad $ средний $ \ quad $ наибольший

покажи ответ

Вопрос с подвохом.Все три резистора имеют одинаковое напряжение. Вот что значит быть параллельным. Все они имеют одинаковое напряжение.

Особый случай — резисторы по 2 доллара параллельно

Из общего уравнения два резистора, включенных параллельно, имеют эквивалентное сопротивление

Ом.

$ \ text R _ {\ text {parallel}} = \ dfrac {1} {\ left (\ dfrac {1} {\ text R1} + \ dfrac {1} {\ text R2} \ right)} $

Мы можем немного поработать алгеброй, чтобы придумать альтернативное выражение с одной дробью.Вместо того, чтобы просто сказать вам ответ, это обряд посвящения в самостоятельную работу с алгеброй в первый раз. Ответ спрятан, так что вы можете попробовать это самостоятельно, прежде чем посмотреть. Посмотрим, сможете ли вы получить выражение для $ \ text R _ {\ text {parallel}} $ в виде одной дроби.

Два резистора параллельно

$ \ text R _ {\ text {parallel}} = \ dfrac {1} {\ left (\ dfrac {1} {\ text R1} + \ dfrac {1} {\ text R2} \ right)} $

Обработайте знаменатель, чтобы очистить дроби. Общий знаменатель — $ \ text R1 \, \ text R2 $.

$ \ text R _ {\ text {parallel}} = \ dfrac {1} {\ left (\ dfrac {1} {\ text R1} \, \ dfrac {\ text R2} {\ text R2} \ right) + \ left (\ dfrac {1} {\ text R2} \, \ dfrac {\ text R1} {\ text R1} \ right)} $

$ \ text R _ {\ text {parallel}} = \ dfrac {1} {\ left (\ dfrac {\ text R2} {\ text R1 \, \ text R2} \ right) + \ left (\ dfrac {\ текст R1} {\ text R1 \, \ text R2} \ right)} $

$ \ text R _ {\ text {parallel}} = \ dfrac {1} {\ left (\ dfrac {\ text R1 + \ text R2} {\ text R1 \, \ text R2} \ right)} $

Переверните большую дробь, чтобы получить уравнение для двух параллельных резисторов,

$ \ text R _ {\ text {parallel}} = {\ dfrac {\ text R1 \, \ text R2} {\ text R1 + \ text R2}} $

Произведение на сумму.Это стоит запомнить.

Особый случай — резисторы равной 2 $, включенные параллельно

Что делать, если два параллельно включенных резистора имеют одинаковое значение. Что эквивалентно $ \ text R _ {\ text {parallel}} $?

Пусть $ \ text R1 = \ text R2 $. Мы назовем его просто $ \ text R $.

$ \ text R _ {\ text {parallel}} = \ dfrac {\ text R \, \ text R} {\ text R + \ text R} = \ dfrac {\ text R \, \ text R} {2 \ , \ text R}

долларов США

$ \ text R _ {\ text {parallel}} = \ dfrac {1} {2} \, \ text

R $

Два идентичных резистора, включенных параллельно, имеют эквивалентное сопротивление, равное половине значения любого резистора.Ток распределяется между ними поровну.

Резюме

Параллельно подключенные компоненты имеют одинаковое напряжение.

Общая форма резисторов, включенных параллельно,

$ \ dfrac {1} {\ text R _ {\ text {parallel}}} = \ dfrac {1} {\ text R1} + \ dfrac {1} {\ text R2} + \ ldots + \ dfrac {1} {\ text {R} _ \ text N}

долларов США

Используйте это уравнение, если у вас параллельно подключено три или более резистора.

Если у вас есть два параллельных резистора, вы можете использовать общую форму или объединить их как произведение на сумму,

$ \ text R _ {\ text {parallel}} = \ dfrac {\ text R1 \, \ text R2} {\ text R1 + \ text R2} $

Эквивалентный резистор $ \ text R _ {\ text {parallel}} $ всегда меньше наименьшего параллельного резистора.

Самый большой ток протекает через наименьший резистор.

Параллельные резисторы | Электрические цепи

17,5 Резисторы параллельные (ЭСАФК)

Когда мы добавляем резисторы параллельно цепи:

  • Есть больше путей для прохождения тока, что обеспечивает разделение тока по различным путям .

  • Напряжение на резисторах то же .Напряжение на батарее в цепи равно напряжению на каждом из параллельных резисторов:

    \ [{V} _ {\ text {battery}} = {V} _ {1} = {V} _ {2} = {V} _ {3} \ ldots \]
  • Сопротивление току уменьшается . Общее сопротивление \ ({R} _ {P} \) дается выражением:

    \ [\ frac {1} {{R} _ {P}} = \ frac {1} {{R} _ {1}} + \ frac {1} {{R} _ {2}} + \ ldots \ ]

При параллельном подключении резисторов начальная и конечная точки для всех резисторов одинаковы.Эти точки имеют одинаковую потенциальную энергию, поэтому разность потенциалов между ними одинакова, независимо от того, что между ними вставлено. Между двумя точками может быть один, два или несколько резисторов, разность потенциалов не изменится. Вы можете игнорировать любые компоненты, находящиеся между двумя точками в цепи, при вычислении разницы между двумя точками.

Посмотрите на следующие принципиальные схемы. Батарея во всех случаях одна и та же, меняется только добавление резисторов между точками, отмеченными черными точками.Если бы мы измерили разность потенциалов между двумя точками в этих схемах, мы бы получили одинаковый ответ для всех трех случаев.

Давайте посмотрим на два параллельно подключенных резистора более внимательно. Когда вы создаете схему, вы используете провода, и вы можете подумать, что измерение напряжения в разных местах на проводах будет иметь значение. Это неправда. Измерение разности потенциалов или напряжения будет отличаться только в том случае, если вы измеряете другой набор компонентов. Все точки на проводах, между которыми нет компонентов цепи, дадут вам одинаковые измерения.

Все три измерения, показанные на рисунке ниже (т. Е. A – B, C – D и E – F), дадут вам одно и то же напряжение. Различные точки измерения слева не имеют компонентов между собой, поэтому нет изменений в потенциальной энергии. То же самое относится и к разным точкам справа. Когда вы измеряете разность потенциалов между точками слева и справа, вы получите одинаковый ответ.

Рабочий пример 7: Напряжения I

Рассмотрим эту принципиальную схему:

Какое напряжение на резисторе в показанной цепи?

Проверьте, что у вас есть и единицы

У нас есть схема с батареей и одним резистором.Нам известно напряжение на батарее. Мы хотим найти это напряжение на резисторе.

\ [{V} _ {\ text {battery}} = \ text {2} \ text {V} \]

Применимые принципы

Мы знаем, что напряжение на батарее должно быть равно общему напряжению на всех других компонентах схемы.

\ [{V} _ {\ text {battery}} = {V} _ {\ text {total}} \]

Есть только один другой компонент схемы, резистор.

\ [{V} _ {\ text {total}} = {V} _ {1} \]

Это означает, что напряжение на батарее такое же, как напряжение на резисторе.

\ begin {align *} {V} _ {\ text {battery}} & = {V} _ {\ text {total}} = {V} _ {1} \\ {V} _ {1} & = \ text {2} \ text {V} \ end {align *}

Рабочий пример 8: Напряжение II

Рассмотрим эту схему:

Какое напряжение на неизвестном резисторе в показанной цепи?

Проверьте, что у вас есть и единицы

У нас есть схема с батареей и двумя резисторами. Нам известно напряжение на батарее и на одном из резисторов.Мы хотим найти это напряжение на резисторе.

\ begin {align *} {V} _ {\ text {battery}} & = \ text {2} \ text {V} \\ {V} _ {B} & = \ text {1} \ text {V} \ end {align *}

Применимые принципы

Мы знаем, что напряжение на аккумуляторе должно быть равно общему напряжению на всех других компонентах схемы, включенных последовательно.

\ [{V} _ {\ text {battery}} = {V} _ {\ text {total}} \]

Общее напряжение в цепи — это сумма напряжений на отдельных резисторах

\ [{V} _ {\ text {total}} = {V} _ {A} + {V} _ {B} \]

Использование зависимости между напряжением на батарее и общим напряжением на резисторах

\ begin {align *} {V} _ {\ text {battery}} & = {V} _ {\ text {total}} \\\\ {V} _ {\ text {battery}} & = {V} _ {1} + {V} _ {\ text {resistor}} \\ \ text {2} \ text {V} & = {V} _ {1} + \ text {1} \ text {V} \\ {V} _ {1} & = \ text {1} \ text {V} \ end {align *}

Рабочий пример 9: Напряжения III

Рассмотрим принципиальную схему:

Какое напряжение на неизвестном резисторе в показанной цепи?

Проверьте, что у вас есть и единицы

У нас есть схема с батареей и тремя резисторами.Нам известно напряжение на батарее и двух резисторах. Мы хотим найти это напряжение на неизвестном резисторе.

\ begin {align *} {V} _ {\ text {battery}} & = \ text {7} \ text {V} \\ {V} _ {\ text {известное}} & = {V} _ {A} + {V} _ {C} \\ & = \ текст {1} \ текст {V} + \ текст {4} \ текст {V} \ end {align *}

Применимые принципы

Мы знаем, что напряжение на аккумуляторе должно быть равно общему напряжению на всех других компонентах схемы, включенных последовательно.

\ [{V} _ {\ text {battery}} = {V} _ {\ text {total}} \]

Общее напряжение в цепи — это сумма напряжений на отдельных резисторах

\ [{V} _ {\ text {total}} = {V} _ {B} + {V} _ {\ text {known}} \]

Использование зависимости между напряжением на батарее и общим напряжением на резисторах

\ begin {align *} {V} _ {\ text {battery}} & = {V} _ {\ text {total}} \\ {V} _ {\ text {battery}} & = {V} _ {B} + {V} _ {\ text {known}} \\ \ text {7} \ text {V} & = {V} _ {B} + \ text {5} \ text {V} \\ {V} _ {B} & = \ text {2} \ text {V} \ end {align *}

Рабочий пример 10: Напряжения IV

Рассмотрим принципиальную схему:

Какое напряжение на параллельной комбинации резисторов в показанной цепи? Подсказка: остальная часть схемы такая же, как и в предыдущей задаче.

Быстрый ответ

Схема такая же, как и в предыдущем примере, и мы знаем, что разница напряжений между двумя точками в цепи не зависит от того, что между ними, поэтому ответ такой же, как и выше \ ({V} _ {\ text {parallel }} = \ текст {2} \ текст {V} \).

Проверьте, что у вас есть и единицы — длинный ответ

У нас есть схема с батареей и пятью резисторами (два последовательно и три параллельно). Нам известно напряжение на батарее и двух резисторах.Мы хотим найти это напряжение на параллельных резисторах \ ({V} _ {\ text {parallel}} \).

\ begin {align *} {V} _ {\ text {battery}} = \ text {7} \ text {V} \\ {V} _ {\ text {известное}} = \ text {1} \ text {V} + \ text {4} \ text {V} \ end {align *}

Применимые принципы

Мы знаем, что напряжение на батарее должно быть равно общему напряжению на всех других компонентах схемы.

\ [{V} _ {\ text {battery}} = {V} _ {\ text {total}} \]

Напряжения складываются только алгебраически для последовательно соединенных компонентов.Параллельно подключенные резисторы можно рассматривать как единый компонент, который включен последовательно с другими компонентами, а затем можно складывать напряжения.

\ [{V} _ {\ text {total}} = {V} _ {\ text {parallel}} + {V} _ {\ text {известное}} \]

Использование зависимости между напряжением на батарее и общим напряжением на резисторах

\ begin {align *} {V} _ {\ text {battery}} & = {V} _ {\ text {total}} \\ {V} _ {\ text {battery}} & = {V} _ {\ text {parallel}} + {V} _ {\ text {known}} \\ \ text {7} \ text {V} & = {V} _ {\ text {parallel}} + \ text {5} \ text {V} \\ {V} _ {\ text {parallel}} & = \ text {2} \ text {V} \ end {align *}

В отличие от последовательного случая, когда мы добавляем резисторы параллельно, мы создаем еще путей , по которым может течь ток.Таким образом, мы уменьшаем на общее сопротивление цепи на !

Взгляните на диаграмму ниже. Слева у нас та же схема, что и в предыдущем разделе, с батареей и резистором. Амперметр показывает ток \ (\ text {1} \) \ (\ text {A} \). Справа мы добавили второй резистор параллельно первому. Это увеличило количество путей (ответвлений), по которым заряд может проходить через цепь — общее сопротивление уменьшилось. Вы можете видеть, что ток в цепи увеличился.Также обратите внимание, что ток в разных ветвях может быть разным.

Общее сопротивление ряда параллельных резисторов НЕ является суммой отдельных сопротивлений, так как общее сопротивление уменьшается с увеличением количества путей прохождения тока. Общее сопротивление для параллельных резисторов равно:

\ [\ frac {1} {{R} _ {P}} = \ frac {1} {{R} _ {1}} + \ frac {1} {{R} _ {2}} + \ ldots \ ]

Давайте рассмотрим случай, когда у нас есть два резистора, включенных параллельно, и выясним, каким будет конечное сопротивление.Эта ситуация показана на диаграмме ниже:

Применяя формулу для общего сопротивления, получаем:

\ begin {align *} \ frac {1} {{R} _ {P}} & = \ frac {1} {{R} _ {1}} + \ frac {1} {{R} _ {2}} + \ ldots \\ & \ text {Резисторов всего два} \\ \ frac {1} {{R} _ {P}} & = \ frac {1} {{R} _ {1}} + \ frac {1} {{R} _ {2}} \\ & \ text {Добавить дроби} \\ \ frac {1} {{R} _ {P}} & = \ frac {1} {{R} _ {1}} \ times \ frac {{R} _ {2}} {{R} _ {2 }} + \ frac {1} {{R} _ {2}} \ times \ frac {{R} _ {1}} {{R} _ {1}} \\ \ frac {1} {{R} _ {P}} & = \ frac {{R} _ {2}} {{R} _ {1} {R} _ {2}} + \ frac {{R} _ {1}} {{R} _ {1} {R} _ {2}} \\ & \ text {Переставить} \\ \ frac {1} {{R} _ {P}} & = \ frac {{R} _ {2} + {R} _ {1}} {{R} _ {1} {R} _ {2} } \\ \ frac {1} {{R} _ {P}} & = \ frac {{R} _ {1} + {R} _ {2}} {{R} _ {1} {R} _ {2} } \\ {R} _ {P} & = \ frac {{R} _ {1} {R} _ {2}} {{R} _ {1} + {R} _ {2}} \ end {выровнять *}

Для любых двух параллельно подключенных резисторов мы теперь знаем, что

\ [{R} _ {P} = \ frac {\ text {произведение сопротивлений}} {\ text {сумма сопротивлений}} = \ frac {{R} _ {1} {R} _ {2}} { {R} _ {1} + {R} _ {2}} \]

Делители тока

Цель

Проверьте, что происходит с током и напряжением в последовательных цепях при добавлении дополнительных резисторов.

Аппарат

  • Аккумулятор

  • Вольтметр

  • Амперметр

  • Провода

  • Резисторы

Метод

  • Подключите каждую цепь, показанную ниже

  • Измерьте напряжение на каждом резисторе в цепи.

  • Измерьте ток до и после каждого резистора в цепи, а также до и после параллельных ветвей.

Результаты и выводы

  • Сравните токи через отдельные резисторы друг с другом.

  • Сравните сумму токов через отдельные резисторы с током до параллельных ветвей.

  • Сравните различные измерения напряжения на параллельных резисторах.

Рабочий пример 11: Параллельные резисторы I

А цепь содержит два параллельно включенных резистора. Резисторы имеют значения сопротивления \ (\ text {15} \) \ (\ text {Ω} \) и \ (\ text {7} \) \ (\ text {Ω} \).

Какое полное сопротивление в цепи?

Проанализировать вопрос

Нам говорят, что резисторы в цепи включены в параллельную цепь и что нам нужно рассчитать общее сопротивление.Значения двух резисторов указаны в правильных единицах, Ом.

Применять соответствующие принципы

Было показано, что полное сопротивление резисторов, включенных параллельно, является произведением сопротивлений, разделенных на сумму. Мы можем использовать

\ [{R} _ {P} = \ frac {{R} _ {1} {R} _ {2}} {{R} _ {1} + {R} _ {2}} \]

У нас всего два резистора, и теперь мы резисторы. В данном случае это:

\ begin {align *} {R} _ {P} & = \ frac {{R} _ {1} {R} _ {2}} {{R} _ {1} + {R} _ {2}} \\ & = \ frac {\ left (\ text {15} \ text {Ω} \ right) \ left (\ text {7} \ text {Ω} \ right)} {\ text {15} \ text {Ω} + \ text {7} \ text {Ω}} \\ & = \ frac {105 {\ Omega} ^ {2}} {\ text {22} \ text {Ω}} \\ & = \ текст {4,77} \ текст {Ω} \ end {align *}

Цитировать окончательный результат

Полное сопротивление параллельно включенных резисторов \ (\ text {4,77} \) \ (\ text {Ω} \)

Рабочий пример 12: Параллельные резисторы II

Мы добавляем третий параллельный резистор в конфигурацию (настройку) в предыдущем примере.Дополнительный резистор имеет сопротивление \ (\ text {3} \) \ (\ text {Ω} \).

Какое полное сопротивление в цепи?

Проанализировать вопрос

Нам говорят, что резисторы в цепи включены в параллельную цепь и что нам нужно рассчитать общее сопротивление. Значение дополнительного резистора указано в правильных единицах, Ом.

Применять соответствующие принципы

Общее сопротивление резисторов, включенных параллельно, было задано как

. \ [\ frac {1} {{R} _ {P}} = \ frac {1} {{R} _ {1}} + \ frac {1} {{R} _ {2}} + \ ldots \ ]

У нас есть три резистора, и теперь мы резисторы.В данном случае это:

\ begin {align *} \ frac {1} {{R} _ {P}} & = \ frac {1} {{R} _ {1}} + \ frac {1} {{R} _ {2}} + \ ldots \\ & \ text {есть три резистора} \\ \ frac {1} {{R} _ {P}} & = \ frac {1} {{R} _ {1}} + \ frac {1} {{R} _ {2}} + \ frac {1 } {{R} _ {3}} \\ & \ text {добавить дроби} \\ \ frac {1} {{R} _ {P}} & = \ frac {1} {{R} _ {1}} \ times \ frac {{R} _ {2} {R} _ {3}} {{R} _ {2} {R} _ {3}} + \ frac {1} {{R} _ {2}} \ times \ frac {{R} _ {1} {R} _ {3} } {{R} _ {1} {R} _ {3}} + \ frac {1} {{R} _ {3}} \ times \ frac {{R} _ {1} {R} _ {2 }} {{R} _ {1} {R} _ {2}} \\ \ frac {1} {{R} _ {P}} & = \ frac {{R} _ {2} {R} _ {3}} {{R} _ {1} {R} _ {2} { R} _ {3}} + \ frac {{R} _ {1} {R} _ {3}} {{R} _ {1} {R} _ {2} {R} _ {3}} + \ frac {{R} _ {1} {R} _ {2}} {{R} _ {1} {R} _ {2} {R} _ {3}} \\ & \ text {переставить} \\ \ frac {1} {{R} _ {P}} & = \ frac {{R} _ {2} {R} _ {3} + {R} _ {1} {R} _ {3} + { R} _ {2} {R} _ {3}} {{R} _ {1} {R} _ {2} {R} _ {3}} \\ {R} _ {P} & = \ frac {{R} _ {1} {R} _ {2} {R} _ {3}} {{R} _ {2} {R} _ {3} + {R} _ {1} {R} _ {3} + {R} _ {2} {R} _ {3}} \\ {R} _ {P} & = \ frac {\ left (\ text {15} \ text {Ω} \ right) \ left (\ text {7} \ text {Ω} \ right) \ left (\ text { 3} \ text {Ω} \ right)} {\ left (\ text {7} \ text {Ω} \ right) \ left (\ text {3} \ text {Ω} \ right) + \ left (\ text {15} \ text {Ω} \ right) \ left (\ text {3} \ text {Ω} \ right) + \ left (\ text {7} \ text {Ω} \ right) \ left (\ text { 15} \ text {Ω} \ right)} \\ {R} _ {P} & = \ frac {315 {\ Omega} ^ {3}} {21 {\ Omega} ^ {2} + 45 {\ Omega} ^ {2} + 105 {\ Omega} ^ { 2}} \\ {R} _ {P} & = \ frac {315 {\ Omega} ^ {3}} {171 {\ Omega} ^ {2}} \\ {R} _ {P} & = \ text {1,84} \ text {Ω} \ end {align *}

Цитировать окончательный результат

Полное сопротивление параллельно включенных резисторов \ (\ text {1,84} \) \ (\ text {Ω} \)

При расчете сопротивления для сложных конфигураций резисторов вы можете начать с любой комбинации двух резисторов (последовательно или параллельно) и рассчитать их общее сопротивление.Затем вы можете заменить их одним резистором с рассчитанным вами общим сопротивлением. Теперь используйте этот новый резистор в сочетании с любым другим резистором и повторяйте процесс, пока не останется только один резистор. В приведенном выше примере мы могли бы просто использовать ответ из первого примера параллельно с новым резистором, и мы получили бы тот же ответ.

Рабочий пример 13: Параллельные резисторы III

Мы добавляем третий параллельный резистор к первой конфигурации (настройке), работающей параллельно.Дополнительный резистор имеет сопротивление \ (\ text {3} \) \ (\ text {Ω} \)

Какое полное сопротивление в цепи?

Проанализировать вопрос

Нам говорят, что резисторы в цепи включены в параллельную цепь и что нам нужно рассчитать общее сопротивление. Значение дополнительного резистора указано в правильных единицах, Ом.

Применять соответствующие принципы

Мы можем поменять местами резисторы, не меняя схему:

Мы уже рассчитали, что полное сопротивление двух резисторов в пунктирной рамке равно \ (\ text {4,77} \) \ (\ text {Ω} \). {2}} {\ text {11,77} \ Omega} \\ & = \ текст {1,84} \ текст {Ω} \ end {align *}

Цитировать окончательный результат

Полное сопротивление параллельно включенных резисторов равно \ (\ text {1,84} \) \ (\ text {Ω} \).Это тот же результат, что и при одновременном сложении всех трех резисторов.

Зарегистрируйтесь, чтобы получить стипендию и возможности карьерного роста. Используйте практику Сиявулы, чтобы получить наилучшие возможные оценки.

Зарегистрируйтесь, чтобы открыть свое будущее

Resistance

Exercise 17.1

Как называется единица сопротивления и каков ее символ?

Решение пока недоступно

Объясните, что происходит с общим сопротивлением цепи, когда резисторы добавляются последовательно?

Решение пока недоступно

Объясните, что происходит с общим сопротивлением цепи, когда резисторы добавляются параллельно?

Решение пока недоступно

Почему батарейки разряжаются?

Решение еще не доступно

Цепь параллельных резисторов и простая схема делителя тока

Формула для параллельного резистора будет нашим основным фокусом здесь.После обсуждения последовательного резистора и делителя напряжения давайте узнаем о параллельных резисторах и делении тока. Параллельный резистор относится к одному из пассивных элементов.

Вы узнаете, включены ли резисторы параллельно или последовательно, просто по их клеммному соединению. Мы называем резисторы подключенными параллельно, если их выводы соединены вместе, соответственно, от резистора к резистору.

Определение параллельной цепи

Этот параллельный резистор отличается от последовательного резистора, где есть только один прямой путь, проходящий через резисторы.В параллельном резисторе будет много путей от головы к голове и от хвоста к хвосту. Исходя из этого термина, параллельный резистор используется как делитель тока, а последовательный резистор — как делитель напряжения.

Поскольку ток разделен на несколько путей или ответвлений, ток для каждой ветви может отличаться друг от друга. Но падение напряжения на каждом резисторе будет одинаковым для каждого другого. Отсюда следует, что для резисторов, включенных параллельно, они могут иметь разный ток через каждый из них, но всегда иметь одинаковое падение напряжения на каждом из них.

Зачем нам нужно изучать формулу для параллельного резистора? Вы должны знать, прочитав этот пост, пока не закончите.

Определение параллельных резисторов — это резистивная схема, в которой резисторы подключены вместе к одним и тем же узлам и образуют более одного пути тока, подключенного к одному источнику напряжения. Если вы все еще не понимаете, как мы делаем параллельную резистивную схему, мы можем найти ее ниже.

Схема параллельной цепи

Пример ниже представляет собой простой параллельный резистор в цепи.Мы используем три резистора: R 1 , R 2 и R 3 . Из объяснения выше, падение напряжения на резисторах при параллельном подключении будет одинаковым для друг друга и равным источнику напряжения.

Следовательно,

Все три резистора соединены между собой между A и B.

В отличие от последовательных резистивных цепей, где эквивалентное сопротивление является суммой всех резисторов, параллельный резистор рассчитывается по-другому.Мы используем величину, обратную сопротивлению (1 / R) для каждого сложенного резистора.

Формула для параллельного резистора

Как мы уже упоминали выше, эквивалентное сопротивление параллельных резисторов является суммой, обратной величине каждого резистора. Если резисторы, соединенные параллельно, имеют одинаковое значение, это будет очень просто.

Как мы уже упоминали выше, мы можем посмотреть на пример ниже:

Если два резистора, подключенных параллельно, имеют одинаковое сопротивление, то эквивалентное сопротивление, R T — это половина сопротивления одного резистора. резистор.Следовательно, эквивалентное сопротивление двух резисторов, соединенных параллельно, если они имеют одинаковое сопротивление, равно R / 2. Если параллельно подключено три резистора, эквивалентное сопротивление равно R / 3 и так далее.


Помимо последовательных резисторов, этот тип цепи является наиболее распространенным типом электрических цепей. Для анализа схемы параллельного резистора мы можем использовать тот же метод для последовательного резистора, используя законы Кирхгофа и закон Ома.


Рассмотрим схему на рисунке.(1),

Рисунок 1. Параллельное подключение резистора

, когда два резистора подключены параллельно и, следовательно, имеют одинаковое напряжение на них.

Из закона Ома

(1)


Применение KCL в узле a дает полный ток i как

(2)


Подставляя уравнение (1) в (2), мы получаем

(3)

, где R eq — эквивалентное сопротивление резисторов, включенных параллельно:

(4)

или

(5)

Следовательно,

Эквивалентное сопротивление двух параллельных резисторов равно произведению их сопротивлений, деленному на их сумму.

Выше представлено простейшее уравнение параллельного резистора, которое мы можем использовать каждый раз, когда оно нам нужно.

Следует отметить, что уравнение (5) работает только для двух параллельно подключенных резисторов.

Мы можем расширить уравнение параллельного резистора в уравнении (4) до общего случая схемы с N резисторами, включенными параллельно. Эквивалентное сопротивление составляет

(6)

Обратите внимание, что R eq всегда меньше, чем сопротивление наименьшего резистора в параллельном соединении.Если R 1 = R 2 =… = R N = R , то

(7)

Например, если 4 резистора с сопротивлением 100 Ом подключены параллельно, их эквивалентное сопротивление составляет 25 Ом.

Имейте в виду, что

Эквивалентное сопротивление параллельных резисторов всегда меньше, чем сопротивление наименьшего резистора, подключенного к этой сети. Следовательно, эквивалентное сопротивление RT будет уменьшаться каждый раз, когда у нас есть дополнительные параллельные резисторы.

Формула проводимости для параллельной цепи

Из-за этого простого, но сложного уравнения параллельного резистора для эквивалентного сопротивления мы узнаем о новом значении, известном как проводимость (G), измеренном в Сименсах (S). Проводимость обратно пропорциональна сопротивлению, где G = 1 / R. После того, как мы получим проводимость, мы преобразуем ее обратно обратно, чтобы получить эквивалентное сопротивление R T параллельных резисторов.

При параллельном подключении резисторов легче использовать проводимость, чем сопротивление.


Из уравнения (6) эквивалентная проводимость для резисторов N составляет

(8)

, где:

Уравнение. (8) утверждает:

Эквивалентная проводимость резисторов, подключенных параллельно, равна сумме их индивидуальной проводимости.

Это означает, что мы можем перерисовать рисунок (1) на (2), где мы заменим сопротивления на проводимости.


Эквивалентные сопротивления параллельных резисторов получаются таким же образом, как эквивалентные сопротивления последовательных резисторов.

Напротив, эквивалентные проводимости последовательных резисторов получаются таким же образом, как эквивалентные сопротивления параллельных резисторов.

Рисунок 2. Эквивалентное сопротивление или проводимость

Следовательно, эквивалентная проводимость G eq последовательно соединенных резисторов N составляет

(9)

Учитывая общий ток i, входящий в узел a на рисунке. (1) с током одинаковые значения напряжения, мы получаем

(10)

Мы определяем параллельные резисторы как резисторы, соединенные между собой между теми же двумя точками.Сам параллельный резистор имеет различные схемы.

Как найти ток в параллельной цепи

Поскольку ток в параллельной цепи зависит от ее сопротивления, теперь мы узнаем, как найти ток в параллельной цепи. Токи I 1 , I 2 ,…, I n , входящие в параллельный путь резисторов, зависят от сопротивления этой ветви. Полный ток I T представляет собой сумму токов в параллельных ветвях.Если сопротивление между ветвями равно, то токи также будут разделены поровну.

Если R 1 = R 2 , то I 1 = I 2 = 0,5 I T . Это означает, что общий ток I T делится поровну на две ветви. Если сопротивление R 1 отличается от сопротивления R 2 , тогда нам нужно рассчитать I 1 и I 2 по-разному.Даже если напряжения на ветвях равны, ток может отличаться по закону Ома.

Для примера параллельной схемы давайте посмотрим на схему ниже и попытаемся найти все токи,

Так как R 1 и R 2 имеют разные значения, тогда токи I 1 и I 2 гарантированно имеют разные значения. Помните один из законов Кирхгофа?


Текущие законы Кирхгофа гласят:

Общий ток, покидающий узел, равен току, входящему в тот же самый узел.

Следовательно,
Полный ток в цепи можно выразить как:

После этого мы будем использовать закон Ома для расчета тока, поступающего в каждую ветвь через резисторы. I 1 — это текущий ввод R 1 , а I 2 — текущий ввод R 2 . Источник напряжения В с имеет 12 В, и мы получаем:

И получаем полный ток,

Чтобы прояснить это, мы используем закон Ома для вычисления I T из В. с и R T .
Общее сопротивление R T равно

Тогда общий ток I T равен

Следовательно, это проясняет наши расчеты.

Мы заключаем, что

Уравнение делителя тока

Комбинируя уравнения. (1) и (10) мы получаем уравнение текущего делителя.

(11)

, который показывает, что полный ток i распределяется между резисторами обратно пропорционально их сопротивлениям.

Это известно как принцип деления тока , а схема на рисунке (1) известна как делитель тока .

Обратите внимание, что больший ток проходит через меньшее сопротивление.

Рисунок 3. Короткое замыкание и разрыв в параллельном соединении

Предположим, что один из резисторов на рисунке (1) равен нулю, скажем, R 2 = 0; поэтому R 2 — это короткое замыкание, как видно на рисунке. (3a).

Из уравнения.(11), R 2 = 0 означает, что i 1 = 0, i 2 = i . Это означает, что весь ток i обходит R 1 и протекает через короткое замыкание R 2 = 0, путь с наименьшим сопротивлением.
При коротком замыкании цепи, как показано на рисунке (3a), обратите внимание, что:

  1. Эквивалентное сопротивление R экв. = 0
  2. Весь ток протекает через короткое замыкание.

Для другого крайнего примера, где R 2 = ∞, то есть R 2 — это разомкнутая цепь, как показано на рисунке. (3b).
Ток по-прежнему течет по пути с наименьшим сопротивлением, R 1 .
Уравнение (11) преобразуется в

(12)

В общем, если делитель тока имеет N проводников, параллельных току источника i , N -й проводник будет иметь ток

(13)

Очень удобно комбинировать резисторы последовательно и параллельно в одно эквивалентное сопротивление R eq .

Такое эквивалентное сопротивление должно иметь те же значения тока и напряжения, что и исходная сеть на выводе.

Примеры параллельного резистора

Давайте рассмотрим приведенный ниже пример для лучшего понимания
Найдите R экв для схемы на рисунке. (4)

Рисунок 4. Пример параллельного резистора

6 Ом и 3 Ом в параллельно

1 Ом и 5 Ом последовательно

2 Ом и 2 Ом последовательно

6 Ом и 4 Ом параллельно

Три резистора последовательно

Сопротивление в Параллельная схема

Сопротивление в параллельной цепи
На примерной схеме, рис. 3-44, два резистора подключены параллельно через 5-вольтовую батарею.Каждый имеет значение сопротивления 10 Ом. Формируется полная цепь, состоящая из двух параллельных путей, и ток течет, как показано.

Рисунок 3-44. — Два равных резистора, включенных параллельно.

Расчет отдельных токов показывает, что через каждое сопротивление проходит половина ампера. Полный ток, протекающий от батареи к переходу резисторов и возвращающийся от резисторов к батарее, равен 1 амперам.

Общее сопротивление цепи можно рассчитать, используя
значения полного напряжения (E T ) и полного тока (I T ).

ПРИМЕЧАНИЕ. С этого момента в примерах задач будут использоваться сокращения и символы для электрических величин.

Дано:


Решение:


Это вычисление показывает, что полное сопротивление составляет 5 Ом; половина номинала любого из двух резисторов.

Поскольку полное сопротивление параллельной цепи меньше, чем у любого из отдельных резисторов, общее сопротивление параллельной цепи не является суммой значений отдельных резисторов, как это было в случае последовательной цепи.Общее сопротивление параллельно включенных резисторов также называется ЭКВИВАЛЕНТНЫМ СОПРОТИВЛЕНИЕМ. ( рэнд, эквивалент ). Термины полное сопротивление и эквивалентное сопротивление используются как синонимы.

Существует несколько методов определения эквивалентного сопротивления параллельных цепей. Лучший метод для данной схемы зависит от количества и номинала резисторов. Для схемы, описанной выше, где все резисторы имеют одинаковое значение, используется следующее простое уравнение:


Это уравнение действительно для любого количества параллельных резисторов с РАВНЫМ ЗНАЧЕНИЕМ.

Пример. Параллельно подключены четыре резистора на 40 Ом. Каково их эквивалентное сопротивление?

Дано:


Решение:


На рис. 3-45 показаны два резистора разного номинала, включенные параллельно. Поскольку показан полный ток, можно рассчитать эквивалентное сопротивление.

Рисунок 3-45. — Пример схемы с неравнопараллельными резисторами.


Дано:


Решение:


Эквивалентное сопротивление цепи, показанной на рисунке 3-45, меньше, чем у любого из двух резисторов (R 1 , R 2 ).Важно помнить, что эквивалентное сопротивление параллельной цепи всегда меньше, чем сопротивление любой ветви.

Эквивалентное сопротивление можно найти, зная отдельные значения сопротивления и напряжение источника. Вычисляя ток каждой ветви, складывая токи ветви для вычисления общего тока и разделив напряжение источника на общий ток, можно найти общий ток. Этот метод хоть и эффективен, но довольно продолжителен. Более быстрый способ найти эквивалентное сопротивление — использовать общую формулу для параллельных резисторов:


Если вы примените общую формулу к схеме, показанной на рисунке 3-45, вы получите такое же значение для эквивалентного сопротивления (2 Ом), что и было. полученный в предыдущем расчете, в котором использовались напряжение источника и полный ток.

Дано:


Решение:


Преобразуйте дроби к общему знаменателю.


Поскольку обе стороны являются взаимными (делятся на одну), игнорируйте обратную функцию.


Формула, которую вы дали для одинаковых резисторов, включенных параллельно


, является упрощением общей формулы для параллельных резисторов


Существуют и другие упрощения общей формулы для параллельных резисторов, которые можно использовать для расчета общей суммы. или эквивалентное сопротивление в параллельной цепи.

ВЗАИМНЫЙ МЕТОД. — Этот метод основан на взятии обратной величины для каждой стороны уравнения. Это представляет собой общую формулу для резисторов, включенных параллельно:


Эта формула используется для определения эквивалентного сопротивления ряда неравных параллельных резисторов. При решении этих задач вы должны найти наименьший общий знаменатель. Если вы не знаете, как найти наименьший общий знаменатель, освежите его в математике, том 1, NAVEDTRA 10069 (серия).

Пример: три резистора подключены параллельно, как показано на рисунке 3-46. Значения резистора: R 1 = 20 Ом, R 2 = 30 Ом, R 3 = 40 Ом. Какое эквивалентное сопротивление? (Используйте обратный метод.)

Рисунок 3-46. — Пример параллельной схемы с разными резисторами ответвления.


Дано:


Решение:

ПРОДУКТ ПРЕОДОЛЕНИЯ СУММЫ. — Удобный способ найти эквивалентное или общее сопротивление двух параллельных резисторов — использовать следующую формулу.


Это уравнение, называемое формулой произведения на сумму, используется настолько часто, что его следует сохранить в памяти.

Пример. Какое эквивалентное сопротивление резисторов на 20 Ом и 30 Ом, подключенных параллельно, как показано на рисунке 3-47?

Рисунок 3-47. — Параллельная схема с двумя неравными резисторами.

Дано:


Решение:


Четыре одинаковых резистора подключены параллельно, каждый резистор имеет омическое значение 100 Ом, каково эквивалентное сопротивление?

Три параллельно соединенных резистора имеют номиналы 12 кОм, 20 кОм и 30 кОм.Какое эквивалентное сопротивление?

Два резистора, соединенных параллельно, имеют номиналы 10 кОм и 30 кОм. Какое эквивалентное сопротивление?

Резисторы при параллельном расчете


Описание как рассчитать резисторы параллельно

Резисторы параллельно

Если ток в цепи распределяется по нескольким резисторам, мы называем это параллельной цепью резисторов.На всех резисторах одинаковое напряжение.

Если известны общий ток и напряжение, мы можем вычислить общий сопротивление по закону Ома по следующей формуле.

\ (\ Displaystyle R_ {ges} = \ гидроразрыва {U} {I} \)

Если значения тока или напряжения неизвестны, мы можем сложить проводимости отдельных резисторов.

\ (\ Displaystyle G_ {ges} = G_1 + G_2 + G_3 \) das entspricht \ (\ displaystyle \ frac {1} {R_ {ges}} = \ frac {1} {R_1} + \ frac {1} {R_2} + \ frac {1} {R_3} \)

Пример с тремя резисторами \ (220 \), \ (330 \) и \ (470 \) Ом

\ (\ displaystyle \ frac {1} {R_ {ges}} = \ frac {1} {R_1} + \ frac {1} {R_2} + \ frac {1} {R_3} = \ frac {1} {220} + \ frac {1} {330} + \ frac {1} {470} = 0.0097 \)
\ (\ Displaystyle R_ {ges} = \ гидроразрыва {1} {0,0097} = 103 Ом \)

Для схемы, в которой два резистора включены параллельно, мы можем перейти к более простой формуле, добавив две дроби

\ (\ displaystyle \ frac {1} {R_ {ges}} = \ frac {1} {R_1} + \ frac {1} {R_2} = \ frac {R_1 + R_2} {R_1 · R_2} \)

Это приводит к

\ (\ Displaystyle R_ {ges} = \ гидроразрыва {R_1 · R_2} {R_1 + R_2} \)

Пример
\ (\ Displaystyle R_ {ges} = \ гидроразрыва {220 · 330} {220 + 330} = 132 \)

Резисторы серии

и параллельные [Analog Devices Wiki]

Цель:

Целью этой лабораторной работы является исследование последовательно и параллельно включенных резисторов.

Серия

и параллельные схемы

Простые схемы, состоящие всего из нескольких компонентов, обычно просты для понимания новичками. Но все усложняется, когда в смесь входит большее количество компонентов. Куда идет ток? Что делают узловые напряжения? Можно ли упростить схему и облегчить понимание? Следующая информация должна помочь.

В этой лабораторной работе мы сначала обсудим разницу между последовательными цепями и параллельными цепями, используя цепи, содержащие самые основные компоненты, резисторы и батареи (или источники напряжения), чтобы показать разницу между двумя конфигурациями.

Прежде чем мы углубимся в объяснение, нам нужно определить, что такое узел схемы. Узел в цепи — это не что иное, как электрическое соединение между двумя или более компонентами. Когда схема изображена на схеме, такой как рисунок 1, узлы представлены проводами (линиями) между компонентами.

Рисунок 1, Пример схемы узла

На схеме изображена схема с 4 резисторами и источником напряжения. Также есть четыре уникальных узла.Цветные узлы (линии) Красный соединяет (+) конец источника напряжения с резистором R 1 , оранжевый соединяет R 1 и R 2 вместе, синий соединяет R 2 с R 3 и R 4 и зеленый соединяет (-) конец источника напряжения с R 3 и R 4 . Обратите внимание, что мы обычно определяем один узел как общий узел, на который ссылаются все остальные узлы, в данном случае это зеленый наземный узел.

Нам также необходимо понять, как ток течет по цепи.Обычный ток течет от более высокого или более положительного напряжения к более низкому или менее положительному напряжению в цепи. Некоторое количество тока будет проходить по каждому пути, который может пройти, чтобы добраться до точки наименьшего напряжения, обычно называемой землей (0 вольт). Используя приведенную выше схему в качестве примера, вот как ток будет течь от положительной клеммы источника напряжения к отрицательной клемме.

Обратите внимание, что в некоторых узлах (например, между R 1 и R 2 ) ток на входе такой же, как на выходе.В других узлах (в частности, трехсторонний переход между R 2 , R 3 и R 4 ) основной (красный) ток разделяется на два разных: фиолетовый ток, протекающий в R 3 и оранжевый ток протекает в R 4 . Также обратите внимание, что токи I R3 и I R4 рекомбинируют в зеленый ток. Это подчеркивает ключевое различие между последовательным и параллельным подключением.

Определение цепей серии

Когда резисторы соединены последовательно (как показано на рисунке 2), вывод одного резистора подключается непосредственно к выводу следующего резистора, без каких-либо других возможных путей, так что весь ток в одном резисторе должен течь в следующий и скоро.

Когда резисторы включены последовательно, они могут быть объединены или объединены в один эквивалентный одиночный резистор с сопротивлением, равным сумме последовательных сопротивлений, , то есть ,

Рисунок 2: Последовательные резисторы, R СЕРИЯ = R 1 + R 2 + R 3 +…

Почему это правда? Закон Ома говорит нам, что напряжение на резисторе равно току через резистор, умноженному на сопротивление.Итак, для приведенной выше последовательной схемы:

Мы знаем, что все резисторы имеют одинаковый ток I S .

Аналогично для остальных трех резисторов так:

Или за вычетом I S :

Таким образом, полное эквивалентное сопротивление — это просто сумма их значений.

Определение параллельных цепей

Когда резисторы включены параллельно (как показано на рисунке 3), все их первые выводы соединены вместе, а все их вторые выводы соединены вместе.

Когда резисторы включены параллельно, они могут быть объединены или объединены в один эквивалентный резистор, значение которого определяется следующим уравнением:

Для двух параллельно подключенных резисторов это упрощает:

Рисунок 3: Параллельные резисторы

Почему это правда? Закон Ома говорит нам, что напряжение на резисторе равно току через резистор, умноженному на сопротивление. Итак, для вышеуказанной параллельной схемы:

Мы знаем, что все резисторы имеют одинаковое напряжение В S .

Ток, подаваемый источником напряжения В S , представляет собой сумму токов в резисторах.

Подставляя четыре резистора, получаем:

Или вычтите V S :

Переставляя сопротивления, получаем полное эквивалентное сопротивление:

Эксперименты

Материалы:

Аппаратный модуль ADALM1000
Макетная плата без пайки и перемычки
Резисторы 3 — 100 Ом
Резисторы 3 — 470 Ом

резисторов в серии:

Поместите три резистора 100 Ом последовательно на беспаечную макетную плату, как показано на рисунке 4.Подключите с помощью перемычек вход CH A к левой стороне первого резистора, а вход CH B к правой стороне того же резистора.

Рисунок 4, последовательно соединенные резисторы

Запустите прибор ALICE M1K Ohm Meter. Здесь показан экран. Программное обеспечение использует известный резистор для проверки неизвестного резистора. ADALM1000 имеет встроенный резистор 50 Ом, который можно использовать для этого. Убедитесь, что выбран параметр Int. Уровень напряжения, который используется для измерения резистора, может быть установлен.Тестирование при максимальном напряжении 5,0 В дает наилучшие результаты для большинства номиналов резисторов. Нажмите Run, и вы должны увидеть что-то подобное с единственным резистором 100 Ом.

Переместите перемычку CH B к правому концу второго резистора, как показано ниже.

Рисунок 5, два резистора последовательно

Омметр должен теперь показать значение для двух последовательно соединенных резисторов или около 200 Ом. Теперь переместите перемычку CH B к правому концу третьего резистора, как показано ниже.

Рисунок 6, три резистора последовательно

Омметр должен теперь показать значение трех последовательно соединенных резисторов или около 300 Ом.

параллельных резисторов:

Теперь замените резисторы 100 Ом на резисторы 470 Ом, как показано на рисунке 7.

Измерение одного резистора 470 Ом

Омметр должен теперь показать значение одиночного резистора или около 470 Ом. Переместите средний резистор 470 Ом так, чтобы он был параллелен резистору справа, как показано ниже.

Измерение двух резисторов 470 Ом, включенных параллельно

Омметр должен теперь показать значение для двух резисторов 470 Ом, включенных параллельно. Соответствует ли измеренное значение формуле для параллельных резисторов?

Переместите третий резистор 470 Ом так, чтобы он был параллелен двум другим резисторам справа, как показано ниже.

Измерение трех резисторов 470 Ом, включенных параллельно

Омметр должен теперь показать значение для трех резисторов 470 Ом, включенных параллельно.Соответствует ли измеренное значение формуле для параллельных резисторов?

Поэкспериментируйте с другими комбинациями резисторов и номиналов, чтобы убедиться, что формулы верны для любого номинала резистора.

Комбинированные схемы

Более сложные соединения резисторов обычно представляют собой просто комбинации последовательного и параллельного соединения. Это часто встречается, особенно если учитывать сопротивление проводов. В этом случае сопротивление провода включено последовательно с другими сопротивлениями, включенными параллельно.

Комбинированная схема может быть разбита на аналогичные части, которые являются последовательными или параллельными, как показано на рисунке 7. На рисунке общее сопротивление может быть вычислено путем соединения трех резисторов друг с другом последовательно или параллельно.

Комбинированные последовательные и параллельные резисторы

R 2 и R 3 соединены параллельно друг другу, поэтому мы знаем, что для этих двух резисторов эквивалентное сопротивление будет:

Комбинированное сопротивление R 2 и R 3 последовательно с R1, поэтому общее эквивалентное сопротивление будет:

Для более сложных комбинированных схем различные части могут быть идентифицированы как последовательные или параллельные, уменьшены до их эквивалентов, а затем уменьшены до тех пор, пока не останется единственное сопротивление.

Добавить комментарий

Ваш адрес email не будет опубликован. Обязательные поля помечены *